pharmacology mcqs for 3rd yr zsmu

131
Pharmacology module-1 Zaporozhye State Medical University , Ukraine, Edited by Rag@Puja 3 rd course 2009-10 [email protected] [email protected] 1. Indicate the type of action of a drug, which is introduced orally, is absorbed into the blood in intestine and acts on the certain organs or tissues: A. General (Resorptive) B. Reflex C. Main D. Local (Topical)

Upload: drraghavender-reddy

Post on 28-Oct-2014

519 views

Category:

Documents


22 download

DESCRIPTION

Pharmacology module1Zaporozhye State Medical University , Ukraine, Edited by Rag@Puja 3rd course 2009-10 [email protected] [email protected]. Indicate the type of action of a drug, which is introduced orally, is absorbed into the blood in intestine and acts on the certain organs or tissues:A. General (Resorptive)B. C. D. E. Reflex Main Local (Topical) Indirect2. The formation of a secondary mediator is obligatory in membraneintracellular mechanism of hormone action. Point out the

TRANSCRIPT

Page 1: Pharmacology MCQS for 3rd yr ZSMU

Pharmacology module-1Zaporozhye State Medical University , Ukraine, Edited by Rag@Puja 3rd course [email protected] [email protected]

1. Indicate the type of action of a drug, which is introduced orally, is absorbed into the blood in intestine and acts on the certain organs or tissues:A. General (Resorptive)B. ReflexC. MainD. Local (Topical)E. Indirect

Page 2: Pharmacology MCQS for 3rd yr ZSMU

2. The formation of a secondary mediator is obligatory in membrane-intracellular mechanism of hormone action. Point out the substance that is unable to be a secondary mediator: A. DiacylglycerolB. GlycerolC. CAMPD. Ca2+E. Inositol-3.4.5-triphosphate

3. What from the specified parameters is an obligatory condition of fast permeation of the medicinal agents through a blood-brain barrier?A. Short half-lifeB. Water solubility C. Lipid-solubility D. Strong binding to proteinsE. Ionized state

4. Which of the drugs readily penetrate into the CNS?A. Ionized drugs B. Polar drugs C. Lipid-soluble D. Binding drugs to plasma proteins E. Hydrophilic drugs with a non-uniform distribution of electrons

5. The patient of 37 years, suffering an obliterating endarteritis of the legs, receives phenylin in a dose of 60 mg/kg. Because of convulsion development (brain trauma is in anamnesis), phenobarbital had been given, after the cancellation of which the nasal bleeding occurred at the patient. The given complication is connected with:A. Conjugation of phenylin with a glucuronic acidB. Inhibition by phenobarbital of enzymes microsomal oxidations in a liverC. Induction by phenobarbital of enzymes microsomal oxidations in a liverD. Oxidative deamination of phenylinE. Aliphatic hydroxylation of phenobarbital

6. Pharmacokinetics includes all of the following processes EXCEPT:A. AbsorptionB. Distribution C. Biotransformation D. Elimination E. Clinical use

Page 3: Pharmacology MCQS for 3rd yr ZSMU

7. How pharmacological activity of a drug with high affinity to plasma protein will be altered at hypoalbuminemia? A. Will slightly decreaseB. Will increaseC. Will greatly decreaseD. Will not changeE. Will disappear

8. The bioavailability is defined as an average level if one is equal:A. 0-20%B. 40-70%C. 100%D. > 70%E. < 40%

9. For prevention remote relapses of 4-days malaria a 42 y.o. patient was given primaquine. On the 3-d day of the treatment with therapeutic doses of the drug patient experienced abdominal pain, cardiac pain, dyspepsia, generalized cyanosis. What is the reason of these adverse effects of the drug? A Cumulation of the therapeutic agentB. Slowing down of drug’s secretion with urineC. Decreasing of activity of liver microsomal enzymesD. Genetic deficiency of Glucose-6-phosphate dehydrogenase E. Potentiation of the drug’s action by other therapeutic agent

10. Which one of the following is TRUE for a drug whose elimination from plasma shows zero-order kinetics?A. The half-life of the drug is proportional to the drug concentration in plasmaB. The amount eliminated per unit time is constantC. The rate of elimination does not depend on the plasma concentrationD. Elimination involves a rate-limiting enzymatic reaction operating at its maximal velocityE. A plot of drug concentration versus time is a straight line

11. Choose the correct statement:A. Weak bases are absorbed efficiently across the epithelial cells of the stomachB. Coadministration of atropine speeds the absorption of a second drugC. Drugs showing large volume of distribution can be efficiently removed by dialysis of the plasma

Page 4: Pharmacology MCQS for 3rd yr ZSMU

D. Stress emotions can lead to a slowing of drug absorptionE. If the volume of distribution for a drug is small, most of the drug is in the extraplasmic space

12. A drug, given in a 100 mg single dose, results in a peak plasma protein concentration of 20 micro gram/ml.The apparent vol. of distribution is: A. 0.5 LB. 1 LC. 2 LD.5LE. 10 L

13. What is the main mechanism of drug absorption in the GIT?A. FiltrationB. Passive diffusion C. Active transport D. PinocytosisE. All above

14. What reactions may occur only in repeated introductions of drugs?A. PotentiationB. CumulationC. ToleranceD. TachyphylaxisE. All above except A

15. How is the drug action, which causes congenital malformations, called?A. Mutagenous actionB. Embriotoxic actionC. TeratogenicD. FetotoxicE. All above

16. How to explain the reduction of effects of salicylates in the interaction with Phenobarbital?A. Inhibition of enzymes of the liverB. Antagonist effectC. TachyphylaxisD. ToleranceE. Activation of enzymes of the liver

17. The breadth of therapeutic action is a range of doses:A. From average therapeutic dose to minimum toxic doseB. From single dose to course doseC. From minimum effective (threshold) dose to minimum toxic dose

Page 5: Pharmacology MCQS for 3rd yr ZSMU

D. From single dose to daily doseE. From minimum effective dose to average toxic dose

18. Give the correct explanation of the term “tolerance”:A. Strong wish of taking the drugB. Amplification of drug action in repeated introductionC. Lowering of drug action in repeated introductionD. Increased sensibility to the drugE. All above

19 What is the acetylation of drugs? A. Conjugation with glucuronic acidB. Oxidation with participation of cytochrome P-450C. Binding to acetic acidD. Hydrolysis of drugsE. Binding to glucuronic acid

20. What is the glucuronidation of drugs? A. Conjugation with glucuronic acidB. Oxidation with participation of cytochrome P-450C. Binding to acetic acidD. Hydrolysis of drugsE. Cyclization of drugs

21. What does the term «tachyphylaxis» mean?A. Strong wish of taking the drugB. Amplification of drug action in repeated introductionC. Lowering of drug action in repeated introductionD. Increased sensibility to the drugE. Rapid decrease of the effect in repeated introduction of the drug within short period of time

22. The cumulation of drugs is possible:A. In decreased reabsorption in kidneyB. In increased secretion in renal tubulesC. In lowering of the glomerular filtrationD. In activation of microsomal enzymes of the liverE. In increase of biotransformation

23. A patient took PO half of the glass of chlorophos (strong inhibitor of the acetylcholinesterase) with the aim of suicide. Besides of gastric lavage, cleansing enema, infusion therapy, the injection of atropine was made. As a result of the signs of the intoxication completely disappeared.Determine the type of interaction between atropine and chlorophos:A. Inhibition of enzymes of the liverB. Antagonist effectC. TachyphylaxisD. ToleranceE. Activation of enzymes of the liver

Page 6: Pharmacology MCQS for 3rd yr ZSMU

24. A patient took the long-lasting course of antibiotic therapy in connection with infectious disease. During treatment he began to complain of skin rash, itching, rhinitis. After the next injection of the antibiotic asphyxia, edema of lips, increase of body to were observed. The doctor abolished the antibiotic therapy and indicated necessary drugs. Determine the type of this pathological condition:A. Inhibition of enzymes of the liverB. Sensibilization C. TachyphylaxisD. ToleranceE. Antagonist effect

25. Which of the following statements result in a doubling of the steady-state concentration of a drug? A. Doubling the rate of infusionB. Maintaining the infusion rate, but doubling the loading doseC. Doubling the rate of infusion and doubling the concentration of the infused drugD. Tripling the rate of infusionE. Quadrupling the rate of infusion

26. Azithromycin, an antibiotic, has an apparent volume of distribution (Vd) of approximately 30 L/kg. The correct interpretation of this information is that Azithromycin is which of the following?A. Effective only when given intravenouslyB. Extensively distributed to sites outside the vascular and interstitial spacesC. Eliminated mainly by renal excretion, without prior metabolismD. Unable to cross the blood-brain or placental barriersE. Not extensively bound to plasma proteins

27. Which of the following administration routes most likely subject a drug to a “first-pass” effect in the liver?A. InhalationB. IntramuscularC. IntravenousD. OralE. Sublingual

28. You are planning to infuse a drug IV at a constant amount per unit time (rate). It has a first order elimination rate constant (kel) of 0.35/h. No loading dose will be given. Approximately how long will it take for blood levels to reach steady state after the infusion begins?A. 0.7 hoursB. 1.2 hours C. 9 hoursD. 24 hoursE. 36 hours

Page 7: Pharmacology MCQS for 3rd yr ZSMU

29. Two drugs act on the same tissue or organ via activation of different receptors, resulting in effects that are qualitatively the opposite of one another. What is the type of drug antagonism?A. ChemicalB. PhysiologicC. DispositionalD. PharmacologicE. Competitive

30. Indicate chemical reaction of drug metabolism which belongs to the phase of synthetic reactions (conjugation):A. DeaminationB. DealkylationC. OxidationD. Glucuronidation E. Reduction

31. What is the mechanism of absorption of a drug substance possessing lipophilic properties?A. Active transportB. Passive diffusion C. FiltrationD. PinocytosisE. Binding to transport proteins

32. Excretion of drugs or their metabolites across the cell membrane occurs by the following mechanism only:A. Passive diffusion B. Active transportC. FiltrationD. PinocytosisE. Simplified diffusion

33. If the drug substance or toxin inhibits synthesis of ATP, what mechanism of penetration through the cell membrane is blocked?A. Passive diffusion B. Active transport C. Filtration D. PinocytosisE. Simplified diffusion

34. What pharmacokientic property is characteristic for lipophilic drug substance?A. It is metabolized in liverB. Low bioavailability in oral introductionC. Quick renal eliminationD. Low permeability across tissue barriersE. Low renal reabsorption

Page 8: Pharmacology MCQS for 3rd yr ZSMU

35. Pharmacological incompatibility of medicines, which is found out at a level of specific effect (for example, administration of adrenomimetics and adreno blockers), is known as: A. PhysicalB. PharmacokineticC. ChemicalD. Pharmacodynamic E. Pharmaceutical

36. Influence of certain unfavorable factors, in particular some medical agents, which precede pregnancy, enlarge risk of a birth of the child with genetic defects. How this action is called?A. Embriotoxic effectB. Mutagenic effectC. Teratogenic effectD. Fetotoxic effectE. Blastomogenic effect

37. It is known that in certain people with genetically determined insufficiency of glucose-6-phosphate dehydrogenase the administration of certain antimalarial agents can cause hemolysis. What is the name of this atypical reaction?A. Idiosyncrasy B. SensibilizationC. Allergic reactionD. TachyphylaxisE. Tolerance

38. In a patient’s liver the processes of detoxification of natural metabolites and xenobiotics is broken. Activity of what cytochrome can be reduced? A. Cytochrome BB. Cytochrome B -1C. HemoglobinD. Cytochrome oxidaseE. Cytochrome P-450

39. Morphine undergoes glucuronidation in the liver. Binding of the medicinal preparation with glucuronic acid during its biotransformation will:A. Increase its pharmacodynamic activityB. Increase its water solubility C. Reduce its water solubilityD. Not change its pharmacodynamic activity E.

40. The patient was given a drug “A”. After a few days the effect of the drug singnificanty dropped and for restoring of the initial effect it was necessary to increase the drug’s dose. What is the name of this phenomenon?

Page 9: Pharmacology MCQS for 3rd yr ZSMU

A. CumulationB. TachyphylaxisC. DependenceD. Tolerance E. Idiosyncrasy

41. At embryonal period metabolism of the medicines is slower that in adults. This peculiarity of fetus pharmacokinetics linked with:A. Functional prematurity of majority of enzymes or their absence B. Higher permeability of histohematologic barriersC. Significant volume of extracellular fluidD. Ability of the skin to absorb and excrete water-soluble medicinesE. Development of organs receptors at different terms

42. A patient who was been suffering from cardiac insufficiency for several months has been taking digoxin on an outpatient basis. At a certain stage of treatment there appeared symptoms of drug overdose. What phenomenon underlies the development of this complication? A. TachyphylaxisB. SensibilizationC. Functional cumulationD. HabituationE. Material cumulation

43. A 36 y.o. man has a craniocerebral trauma. Objectively: diminished breath sounds, thready pulse, and no reflexes. What way of pyracetam introduction will be the most appropriate in this case?A. InhalationB. SubcutaneousC. RectalD. Intravenous E. Peroral

44. A patient, who has been treating with Clophelinum (Clonidine) due to arterial hypertension, has taken an alcoholic drink that has caused the sharp inhibition of the CNS. What is the name of this interaction? A. Potentiation B. SummationC. CumulationD. IntoxicationE. Idiosyncratic reaction

45. A patient who has been treated with diazepam on account of neurosis complains of toothache. Doctor administered him an analgetic, but its dose was lower than average therapeutic dose. What phenomenon did the doctor take into account while prescribing the patient an underdose?A. ToleranceB. Cumulation

Page 10: Pharmacology MCQS for 3rd yr ZSMU

C. SummationD. Potentiation E. Drug dependence

46. During surgical operation with administration of a myorelaxant (neuromuscular blocker), a disturbance of breathing developed. Administration of proserine (neostigmine) has improved patient’s condition. What is the name of this type of drug interaction?A. IncompatibilityB. Antagonism C. TachyphylaxisD. SynergismE Cumulation

47. Doctor prescribed 5% ephedrine nasal drops to the patient suffering from acute rhinitis. After the first introduction of the drops nasal breathing of the patient was improved. He started to put the drops every 30 min. As a result there was sharp weakening of the effect of the preparation. Doctor stated appearance of tachyphylaxis to often introduction of the drops. Indicate the reason of appearance of tachyphylaxis:A. Exhaustion of substrate through which ephedrine acts B. Adaptation of cell receptorsC. Increase of activity of hepatic enzymesD. Increase of excretion of ephedrineE. Blockade of receptors

48. Indicate the number of introduction of a drug per day if its half-life period (T1/2) is 20 hours:A. 2 times a dayB. 3 times a dayC. 4 times a dayD. Once dailyE. Every 12 hours

49. What is half-life period (T1/2) of a drug?A. Blood plasma volume, which is cleared from the drug in a period of timeB. Period during which concentration of a drug in blood plasma is decreased by 50% C. Period of complete excretion of a drug from the organismD. Speed of excretion of the drug via kidneysE. Ratio between speed of excretion and blood plasma concentration of a drug

50. Benzylpenicilline natrii salt is injected to the patient with pneumonia in a doze 500,000 UA 6 times per day. Just after an injection the patient developed fever, spasms, loss of consciousness. What has happened to the patient?

Page 11: Pharmacology MCQS for 3rd yr ZSMU

A Anaphylactic shockB. IdiosyncrasyC. TachyphylaxisD. CumulationE. Tolerance

51. The patient suffering from epilepsy had been receiving daily 0,2 g of phenobarbitale continuously. Recently attacks became more frequent, suppressed mood is observed. Which process became the cause of deterioration of the patient’s state?A. Liver monooxygenase enzymes system induction.B. Liver monooxygenase enzymes system inhibition.C. Lipolysis activation.D. Gluconeogenesis activation.E. Glycolysis inhibition.

52. Ambulance has been called to a 22 y.o. man, because of bronchial asthma attack. What way of salbutamole administration is the most appropriate in this case?A. IntravenousB. IntramuscularC. InhalationD. SubcutaneousE. Sublingual

53. A stenocardia attack happened with a 48 y.o. man. The doctor has prescribed him a pill of nitroglycerine sublingually. Why has the doctor chosen a sublingual method of the medicine administration?A Bad intestinal absorptionB Drug elimination by gastric juiceC Effect develops fasterD. It is activated by salivaE. Only because it damages the liver

54. Phenobarbital was prescribed to a 54 y.o. man, suffering from insomnia. The sleep rhythm has been normalized. But gradually, during 2 months, the effect of a medicine has decreased and insomnia reoccured. What factor causes the reduction of somnifacient medication effect?A.Bad solubilityB.ToleranceC.Bad absorption in the stomachD.Accumulation in lipidsE.Tachyphylaxis

55. During the embryonic period metabolism of medications happens considerably slower than in an adult organism. The specified feature of an embryo pharmacokinetics is caused, first of all, by the following:A.The functional imperfection of enzymes, predominantly by their absence

Page 12: Pharmacology MCQS for 3rd yr ZSMU

B The big permeability of histohematologic barriersC Essential volume of extracellular liquidD. Ability for absorbtion and allocation of water-soluble drugs by the skinE. „Maturing” of receptors in organs in different terms

56. It is known,that in people with genetically caused insufficiency of glucose-6-phosphate dehydrogenase enzyme of red blood cells, lysis of stated cells may develop as a reaction to some antimalaric medications injection. How is such untypical reaction to medications designated?A.SensibilisationB.IdiosyncrasyC.Allergic reactionD.TachyphylaxisE.Tolerance

57. How can the fact that for tuberculosis treatment the isoniazide doze is selected individually with the obligatory control of concentration of the medication in urine after its first application be explained?A.Development of haemolytic anaemiasB.Hyperglycemia after drug administrationC.Development of renal insufficiencyD.Irritating effect of the drugE.Genetically caused methylation of the drug

58. The patient with hypertension has been taking an antihypertensive medications for a long time, but suddenly he has stopped doing it. After that patient’s condition has worsened, hypertensive crisis has developed. What kind of a collateral action is it?A CumulationB Syndrome of a cancellation C ToleranceD SensibilisationE Addiction

59. During the visit of a stomatologist, an attack of bronchospasm has developed in the patient who has been treated by 5 % efedrine hydrochloride solution injection. In 20 min the attack reoccured. The additional injection of efedrine had no effect due to tachyphylaxis. What mechanism underlies this phenomenon?A Exhaustion of the noradrenaline depot in presynaptic endingsB Inhibiton of adrenal receptorsC Activation of adrenal receptors D Material cumulation of the drugE Induction of microsomal enzyme system by the liver

Page 13: Pharmacology MCQS for 3rd yr ZSMU

60. A long course of treatment with cyancobalamine was administered IM to the patient with megaloblastic anemia developed after gastroectomy. What advantage does a parenteral way of administration have over oral?A The drug is quickly deducedB The drug is quickly absorbedC The drug circulates in blood for a longer timeD The drug does not disintegrate in the liverE.This way is effective in absence of gastromucoprotein

61. Which of the following conditions is NOT characteristic of cocaine over dosage?A. Dilation of the pupilB. Euphoria C. Tachycardia D. Peripheral vasodilationE. Hallucinations

62. A patient was delivered to the surgical department in connection with acute appendicitis. He was proposed to make appendectomy under the local anesthesia. But from his medical history was known that he had severe allergic reactions to drugs. Choose the drug the most preferable for the infiltration anesthesia in this situation:A. NovocaineB. LidocaineC. AnesthesineD. DicaineE. Any from indicated above

63. A patient was delivered to the ophthalmologic department with trauma of the eye as a result of hitting with metallic shaving. Edema of the eye, significant hyperemia, hyperlacrimation, pain were observed. Choose the local anesthetic to use topically for removal of the foreign body:A. Procaine (Novocaine)B. TrimecaineC. AnesthesineD. DicaineE. Ultracaine

64. A 23-year-old woman with red and itchy eczematous dermatitis visits a doctor office. She had a dental procedure one day earlier with administration of a local anesthetic. There were no other findings, although she indicated that she had a history of allergic reactions. Which of the following drugs is most likely involved?A. LidocaineB. UltracaineC. BupivacaineD. Novocaine (Procaine)E. Trimecaine

Page 14: Pharmacology MCQS for 3rd yr ZSMU

65. A patient with renal failure has a periodic hemodialysis while awaiting a transplant. Between dialysis sessions we want to reduce the body’s phosphate load by reducing dietary phosphate absorption and removing some phosphate already in the blood. Which compound shall we administer orally?A. Aluminum hydroxideB. Magnesium hydroxideC. Sodium bicarbonateD. TanninE. Bismuth subnitras

66. Bismuth salts are thought to be effective adjuncts in managing, if not healing, refractory gastric ulcer because they have bactericidal properties against:A. Staphylococcus aureusB. Clostridium difficileC. E.coliD. Helicobacter pylori E. Bactericides fragilis

67. Indicate the mechanism of action of local anesthetics:A. Formation of albuminates with tissue’s proteinsB. Blockade of Na+-channels C. Blockade of M-cholinoceptorsD. Inhibition of non-specific excitatory systems of CNSE. Blockade of (-adrenoceptors

68. As a result of the influence of terminal anesthesia which part of the skin and mucus membranes are affected:A. Sensory nerve endings B. EpidermisC. Subcutaneous fatty tissueD. Walls of capillariesE. Derma

69. Indicate the principle of action of covering drugs:A. Formation of protective layer on the mucous membranes B. Blockade of mucous membranes receptorsC. Coagulation of proteins of superficial layer of mucous membraneD. Formation of complexes with toxic agentsE. Stimulation of regenerative processes

70. For anesthesia during tooth extraction solution of novocaine(procaine) has been used. Why it was not introduced into gingival tissue, but in the projection of nerve fiber path? Because of:A. Transmission of pain impulses is blocked B. Alteration of tissue pH in anesthesia areaC. Depression of axonal transportD. Disturbance of action potential formation of the pain receptorsE. Increasing of excitability of the pain receptors

Page 15: Pharmacology MCQS for 3rd yr ZSMU

71. Why novocaine is not used for terminal anaesthesia?A Is poorly absorbed through normal skin surface and mucous membrane B. Doesn’t cause covering actionC It is fast absorbed and inhibits the CNSD. Irritates mucous membraneE. Activates M-cholinoceptors

72. Indicate main effect of the local anesthetics:A. Eliminate all kinds of sensibility due to blockade of action potential B. Selective relieve of pain sensibility in local actionC. Decrease of excitability of nerve endingsD. Decrease of excitability and conductivity of the afferent NSE. Eliminate all kinds of sensibility due to paralysis of the CNS

73. Indicate the main effect of astringent drugs.A. Decrease of nerve ending sensibility B. Reduction of hyperemia of mucous membranes due to vasoconstrictionC. Relief of painD. Decrease of synthesis of histamineE. Decrease of gland secretion

74. What drugs from the group of local anaesthetics are not used together with sulfonamides?A. Novocaine (procaine) B. SovcainumC. LidocaineD. TrimecaineE. Ultracaine

75. Determine the drug, which is used for all types of anaesthesia.A. Novocain (procaine)B. Anesthesine (benzocaine)C. Lidocaine D. TrimecaineE. Dicaine (tetracaine)

76. A patient with increased sensitivity to sulfonamide needs tooth extraction. What local anesthetic has to be used?A. Anesthesine (benzocaine)B. Dicaine (tetracaine)C. Lidocaine D. CocaineE. Novocainum (procaine)

77. Injection of a local anesthetic has to be given to a patient for tooth extraction. What drug from listed below is to be chosen?A. CocaineB. Dicaine (tetracaine)

Page 16: Pharmacology MCQS for 3rd yr ZSMU

C. Anesthesine (benzocaine)D. Lidocaine E. Ketamine

78. This agent is poorly soluble in water, so it is used for superficial anesthesia only in the form of ointment, paste and powder. What is this drug?A. SovcainumB. Novocainum (procaine)C. PyromecaineD. TrimecaineE. Anesthesine (benzocaine)

79. What drug has to be added to lidocaine solution to prolong its action?A. Adrenaline B. CaffeineC. Analginum (methamizole)D. AtropineE. Anaprilinum (Propranolol)

80. What is the main indication for adsorbing drugs use?A. DiarrheaB. Hypoacidic gastritisC. Decrease in trypsin activityD. Decrease in bile secretionE. Intoxication

81. A nurse used mustard plaster with water of more than 600C to and applied it on patient’s back. In 30 min she found that patient’s skin under the mustard plaster did not get red. What is the reason for absence of mustard plaster effect?A. Inactivation of mirosine B. Inactivation of choline esteraseC. Activation of mirosineD. Inactivation of monoaminooxydaseE. Activation of methyltranspherase

82. It is necessary to carry out local anesthesia by lidocaine at extraction of a foreign body from an eye. What is the action mechanism of this drug?A. Blocks Na+ channels B. Breaks transit of K+ through the membraneC. Decreases dehydrogenases activityD. Activate K+ channelsE. Activate Na+ channels

Page 17: Pharmacology MCQS for 3rd yr ZSMU

83. A farmer has been delivered to hospital with a sharp pain in his eye. What local anesthetic will cause the strongest anesthesia at terminal anesthesia?A. Novocaine (Procaine)B. LidocaineC. TrimecaineD. Dicaine (Tetracaine) E. Anaesthesine (Benzocaine)

84. Which of the following drugs incorrectly matches a cholinergic agonist with a pharmacologic action?A. Proserine: stimulates atonic bladderB. Carbachol: induces release of epinephrine from the adrenal medullaC. Acetylcholine: decreases heart rate and cardiac outputD. Pilocarpine: reduces intraocular pressureE. Physostigmine: decreases intestinal motility

85. Which of the following signs is NOT an expected symptom of poisoning with Neostigmine (Proserine)?A. Increase in skeletal muscle tonusB. Increased bronchial secretionsC. MiosisD. TachycardiaE. Convulsions

86. Select the most appropriate drug that depolarizes neuromuscular end plate:A. Dithylinum B. DiplacinC. TubocurarineD. MellictinE. Pipecuronium (Arduan)

87. Select the most appropriate drug that reverses the effects of nondepolarising blockers: A. Dithylinum B. Proserine (Neostigmine) C Fresh blood transfusionD ScopolamineE. Ipratropium bromide

88. Select the most appropriate drug to treat the poisoning with depolarising blockers: A. Dithylinum B. Neostigmine C Fresh blood transfusion

Page 18: Pharmacology MCQS for 3rd yr ZSMU

D ScopolamineE. Ipratropium bromide

89. Which ONE of the following drugs would be useful in the long-term treatment of myasthenia gravis?A. ArduanB. AtropineC. Proserine (Neostigmine)D. ScopolamineE. Lobeline

90. A 40-year-old male farm worker is brought to the emergency room. He was found confused in the orchard and since then has lost consciousness. His heart rate is 45 and his BP is 80/40 mm Hg. He is sweating and salivating profusely. Which of the following treatments is indicated? A. GalantamineB. Norepinephrine C. ProserineD. Atropine sulfateE. Ipratropium bromide

91. A patient of 20 years old was delivered to a hospital with sharp pain in the abdomen, vomiting, impeded breathing, and dyspnoea. During the patient’s examination the following data were revealed: the skin is moist, hypersalivation, miosis, bradycardia muscular twitching. From the anamnesis it is known that the patient is a toxoman, he breathed in the aerosol for extermination of insects. What mechanism of action has the agent, which has caused poisoning?A. M-cholinoblockingB. N-cholinoblockingC. M-cholinomimeticD. N-cholinomimeticE. M, N-cholinomimetic

92. The patient with complaints of constant dry mouth addressed a dentist. The doctor made the diagnosis «xerostomia». To normalize the secretion of saliva he administered Pilocarpine 5-6 drops 3 times a day. But the patient violated the instructions and took the drug without following the indicated dosage. Soon besides the disappearance of mouth dryness he felt cramps, diarrhea, increased bronchial secretion, sweating, tearing, bradycardia, constriction in the chest, involuntary urination, hypotension. With the help of what drug can the side effects be easily eliminated?A. AceclidineB. CytitonC. Lobeline

Page 19: Pharmacology MCQS for 3rd yr ZSMU

D. AtropineE. Proserine

93. The effects of tubocurarine can be antagonised by:A. PilocarpineB. AceclidineC. CytitonD. Neostigmine (Proserine)E. Lobeline

94. All of following side effects are seen with Ipratropium bromide except:A. Urinary retentionB. Dryness of mouthC. Scratching in trachea D. Miosis E. Glaucoma

95. The sub-type of cardiac muscarinic receptor is predominantly: A. M1B. M2 C. M3D. M1 and M3E. M1, M2 and M3

96. Which of the following anticholinergic is claimed to act selectively on bronchial muscle?A. Isadrine B. Ipratropium bromideC. SalbutamoleD. PirenzepineE. Ephedrine

97. Physostigmine (Eserini sulfas) which is used mainly for its parasympathomimetic effects, inhibits:A. Tyrosine hydroxylaseB. AcetylcholinesteraseC. Catechol-O-methyltransferase (COMT)D. Monoamine oxidase (MAO)E. DOPA decarboxylase

98. A patient with renal colic has been administered a spasmolytic agent from M-cholinoblockers. Specify this drug.A. Platiphylline

Page 20: Pharmacology MCQS for 3rd yr ZSMU

B. EphedrineC. PirenzepineD. GalanthamineE. No-spa

99. A patient receives Aceclidine after abdominal surgery. His heart rate falls slightly and she experiences some wheezing. These pulmonary and cardiac responses represent or reflect:A. Expected side effect B. IdiosyncrasyC. Parasympathetic ganglion activationD. Reflex (baroreceptor) suppression of cardiac rateE. Undiagnosed asthma

100. We give an ‘effective dose’ of atropine to a person who is poisoned with an acetylcholinesterase inhibitor. Which structure will continue to be overacted by the excess acetylcholine after this drug is given?A. Airway smooth muscleB. S-A node of the heartC. Salivary and lacrimal glandsD. Skeletal muscleE. Vascular smooth muscle

101. Which is a muscarinic receptor-blocking drug that is administered by inhalation to cause bronchodilation for patients with emphysema [chronic obstructive pulmonary disease (COPD)]?A. Platiphyllini hydrotartrasB. Ipratropium bromideC. PancuroniumD. PilocarpineE. Salbutamole

102. A patient with a history of asthma experiences significant bronchoconstriction and urticaria, and histamine is a main mediator in these responses. Which of the following drugs may pose extra risk for this patient – not because it has any bronchoconstrictrictor effects in its own right, but because it quite effectively releases histamine from mast cells?A. AtropineB. Neostigmine sulfate C. PropranololD. PancuroniumE. d-Tubocurarine

Page 21: Pharmacology MCQS for 3rd yr ZSMU

103. A dentist prescribed an agent stimulating salivation to a patient with xerostomia. Indicate the drug.A. PilocarpineB.Dithylinum (suxamethonium)C.ArmineD.ScopolamineE.Atropine

104. A patient had to go through an operation. Doctors introduced him dithylinum (listenone) and performed intubation. After the end of operation and cessation of anesthesia the independent respiration wasn’t restored. Which enzyme deficit prolongs the action of muscle relaxant?A. Succinate dehadrogenaseB. K-Na-adenosine triphosphataseC. CarbanhydraseD. Pseudocholinesterase E. N-acetyltransferase

105. A patient with fracture of his lower jaw was admitted to the maxillofacial department. It was decided to fix his bones surgically under anesthetic. After intravenous introduction of muscle relaxant there arose short febrillar contractions of the patient’s facial muscles. What muscle relaxant was applied? A. DiazepamB. Tubocurarin chlorideC. Pipecuronium bromideD. Dithylinum E. Melictine

106. During surgical operation with administration of a myorelaxant (neuromuscular blocker), a disturbance of breathing developed. Administration of proserine (neostigmine) has improved patient’s condition. What is the name of this type of drug interaction?A. IncompatibilityB. Antagonism C. TachyphylaxisD. SynergismE. Cumulation

107. Introduction of a pharmaceutical substance to an experimental animal resulted in reduction of salivation, pupil mydriasis. Next intravenous introduction of acetylcholine didn’t lead to any significant changes of heart rate. Name this substance: A. ProserineB. AdrenalineC. Propranolol

Page 22: Pharmacology MCQS for 3rd yr ZSMU

D. Atropine E. Salbutamol

108. The doctor has prescribed Cyclodolum for Parkinson disease treatment. What is the mechanism of anti parkinsonian action of the drug?A. M cholinomimetic actionB. M cholinoblocker action C. Stimulation of dopamine receptorsD. Blockade of dopamine receptorsE. (-Adrenoblocker action

109. The glaucoma is diagnosed in 43 y.o. patient for the first time. For the treatment doctor has prescribed an anticholinesterase agent in eye drops. Indicate this agent:A. GalantamineB. PhysostigmineC. PilocarpineD. DipiroximE. Atropine

110. After use of neostigmine in the patient with myasthenia, such symptoms as nausea, diarrhea, twitching of muscles of tongue and skeletal muscles have appeared. What agent can abort the intoxication? A. PhysostigmineB. MethacineC. Mesatone (Phenylephrine)D. IsadrineE. Pyridostigmine bromide

111. M-cholinoblockers have quite wide application in modern clinical practice. What case are M-cholinoblockers contraindicated at?A. Renal colicB. Atony of intestine C. Bronchial asthmaD. Liver colicE. Gastric ulcer

112. During reposition of the bones at a patient with forearm trauma dithylin (succinylcholine) was used. Complete restoration of muscular tonus and functions took more than an hour. What is the reason of the significant prolongation of curare-like action of the agent? A. Genetically determined deficit of butyrylcholinesteraseB. Yielding of an active metaboliteC. Genetically determined deficit of monoamine oxidaseD. Inhibition of microsomal oxidation

Page 23: Pharmacology MCQS for 3rd yr ZSMU

E. Genetically determined deficit of hydroxylase

113. During surgical operation a patient has developed symptoms of dithylin (succinylcholine) overdosage. What treatment is necessary to provide to the patient?A. Administration of anticholinesterasesB. Administration of M-cholinoblockersC. Administration of ganglioblockersD. Blood transfusionE. Administration of N-cholinoblockers

114. The patient with complaints on dryness in mouth, photophobia and visual disturbances is delivered into an admission department. He has also redness and dryness of skin, pupils’ dilation and tachycardia. At the further inspection, the following diagnosis has been established: poisoning by Belladonna alkaloids. What agent from given medicines is wise to use? A. ArmineB. PilocarpineC. Proserine (Neostigmine)D. DipiroximE. Diazepam

115. A 63 y.o. patient has got an injection of pentamine for hypertensive crisis. What is the action mechanism of the drug?A. Stimulation of M-cholinoceptorsB. Inhibition of M-cholinoceptorsC. Inhibition of M-,N-cholinoceptorsD. Stimulation of N-cholinoceptorsE. Inhibition of N-cholinoceptors

116. At complex narcosis (general anesthesia) protocol during stomach resection, a patient has got an injection of tubocurarine chloride as a myorelaxant (neuromuscular blocker). What agent – antagonist should be administered to the patient for restoration of his self-driven respiration?A. Proserine (Neostigmine)B. Dithyline (Succinylcholine)C. AtropineD. EtimizolE. Benzohexonium

117. Atropine is prescribed to the patient for the removing of intestinal colic. What from the named diseases can be a contraindication in this case? 2003A. HypotensionB. Glaucoma

Page 24: Pharmacology MCQS for 3rd yr ZSMU

C. Sinus bradycardiaD. Bronchial asthmaE. Giddiness (Dizziness)

118. A ganglioblocker - Pentamine has been administered to the patient with hypertensive crises. What effects should doctor be worry about after the drug’s injection?A. Suppression of respirationB. DiarrheaC. Rebound syndromeD. Infringement of taste sensationE. Orthostatic (postural) hypotension

119. A M1-cholinoblocker has been administered to the patient for the treatment of gastric peptic ulcer. Indicate this agent:A. Pirensepine (Gastrozepine)B. ScopolamineC. Ipratropium bromideD. MethacineE. Platiphylline

120. Curare-like myorelaxants (neuro-muscular blockers) are used in clinical practice for decreasing and cessation of impulse transmission from nerve ending to muscular fiber. What is the mechanism of action of this group?A. Blockade N-cholinoceptors of postsynaptic membrane of neuromuscular junctionB. Inhibition of acetylcholinesteraseC. Inhibition of Na+/ K+ pumpD. Reduction of neurotransmitter release into synaptic cleftE. Blockade of Ca2+ inward through presynaptic membrane

121. After utilization of ophthalmic drops patient with glaucoma has miosis, myopia, and drop of intraocular pressure. Which pharmacologic group can cause such effects?A. M-cholinomimeticsB. M-cholinoblockersC. N-cholinomimeticsD. GanglioblockersE. Adrenoblockers

122. The victim has been delivered to the traumatologic emergence station by an ambulance with the diagnosis of closed fracture of an average third of the hip with dislocation of bone fragments. To reposit bone fragments, 10 ml of a 2% Ditiline solution was injected

Page 25: Pharmacology MCQS for 3rd yr ZSMU

IV whereupon a continuous apnea and myorelaxation has developed. What enzyme deficiency causes such collateral action?A. N-acetyltranspherase.B. Uridinediphosphoglucuronosyl transferase.C. Glucose-6-phosphate dehydrogenase.D. Methemoglobine reductaseE. Pseudocholine esterase

123. A patient 50 y.o. with complaints of heart palpitation, pain in the heart area, headache, vertigo was delivered to a hospital. During the examination angina pectoris, disorders of cardiac rhythm (paroxysmal tachycardia, atrium fibrillation) and arterial hypertension (180/90 mm Hg) were revealed. Make the rational choice of the drug in this situation.A. Propranolol (Anapriline)B. Epinephrine C. Ephedrine D. NeostigmineE. Reserpine

124. A 60 y.o. asthmatic man comes in for a check-up and complains that he is having some difficulty in «starting to urinate». Physical examination indicates that the man has a BP of 160/100 mm Hg and a slightly enlarged prostate. Which of the following medications would be useful in treating both of these conditions?A. PhentolamineB. ReserpinC. EphedrineD. DoxazosinE. Propranolol

125. A doctor diagnosed hypoglycemic coma in a patient with diabetes mellitus and administered glucose solution IV to him. Patient’s condition improved. What drug can be used additionally as a chemical antagonist of insulin?A. DopamineB. MesatonumC. Strophanthin

D. AdrenalineE. Noradrenaline

126. Anaphylactic shock has delivered in a patient after novocaine (procaine) injection. What agent suppresses histamine release from mast cells and eliminates main symptoms of anaphylactic shock?A. KetotifenB. Cromolin natrium

Page 26: Pharmacology MCQS for 3rd yr ZSMU

C. AdrenalineD. MesatonumE. Noradrenaline

127. A 38-year-old female has taken unknown drug for decreasing high BP. After its administration systolic pressure decreased but tachycardia appeared. What was the patient administered?A. AtenololB. Reserpine

C. PhentolamineD. PropranololE.Prasosine

128. A patient has spasm of smooth muscles of the bronchi. Using of what activators will be physiologically reasonable for emergency treatment?A. ±adrenoceptorSB.D-receptorsC. N-cholinoceptorSD.Beta adrenoreceptorsE.alfa n beta adrenoreceptors

_1_2_9_._ _A_ _2_4_-_y_e_a_r_-_o_l_d_ _p_a_t_i_e_n_t_ _h_a_s_ _b_r_o_n_c_h_i_a_l_ _a_s_t_h_m_a_._ _F_o_r_ _t_h_e_ _t_r_e_a_t_m_e_n_t_ _o_f_ _a_s_t_h_m_a_ _a_t_t_a_c_k_ _a_ _d_o_c_t_o_r_ _h_a_s_ _p_r_e_s_c_r_i_b_e_d_ _s_a_l_b_u_t_a_m_o_l_._ _W_h_a_t_ _i_s_ _t_h_e_ _m_e_c_h_a_n_i_s_m_ _o_f_ _c_u_r_a_t_i_v_e_ _a_c_t_i_o_n_ _o_f_ _t_h_e_ _a_g_e_n_t_?_A. ±_- and ²_-receptors stimulationB. ²_2 adrenoreceptors blockade

C. ²_2 adrenoceptors stimulationD. ²_1 adrenoreceptors blockadeE. ²_1 adrenoreceptors blockade

130. Alpha-1 receptor actions are mediated through: A. AAMPB. AGMP

C.Ca2+ions D.ADPE. K+ Channels

131. With excitation of which receptors are the following effects associated dilation of blood vessels, decrease of tonicity of the bronchial muscles and myometrium,stimulation of glycogenolysis?A. ±1- adrenoceptors

Page 27: Pharmacology MCQS for 3rd yr ZSMU

B. ±2- adrenoceptorsC.Beta1D.Beta 2E.Beta3

132. An ambulance was called for a 22 y.o. patient with status asthmatics. What route of adrenaline administration will be the most appropriate in this case? A. IntravenousB. InhalationC. SubcutaneousD. SublingualE. Intramuscular

133. All the following drugs are used topically in the treatment of chronic wide-angle glaucoma. Which one reduces intraocular pressure by decreasing the formation of the aqueous humor?A. TimololB. PilocarpineC. PhysostigmineD. Neostigmine E. Aceclidine

134. A 40 y.o. patient complaints of intensive heartbeats, sweating, nausea, vision impairment, arm tremor, hypertension. From his anamnesis: 2 years ago he was diagnosed with pheochromocytoma. What hypotensive agent should be administered?A. CaptoprilB. Prazosine C. DrotaverineD. NifedipineE. Platophylline

135. A patient with II stage hypertension has been taking one of hypotensive medications for the purpose of treatment. After a time AP decreased, but the patient started complaining of flaccidity, sleepiness, indifference. A bit later he felt stomach pain. He was diagnosed with ulcer. What hypotensive medication has the patient been taking?A. FurosemideB. VerapamilC. Reserpine D. DibazoleE. Captopril

Page 28: Pharmacology MCQS for 3rd yr ZSMU

136. A patient ill with bronchial asthma didn’t inform his doctor that he had attacks of stenocardia. Doctor administered him a medication, which taking resulted in less frequent attacks of bronchial asthma, but stenocardia attacks became more frequent. What medication was administered? A. SalbutamolB. AminophyllineC. Isadrin D. Cromolin sodiumE. Phenotherol

137. A patient with bronchial asthma has been given 0.5% solution of isadrinum (isoproterenol). Bronchospasm was removed, but the patient started to complain on a pain in cardiac area. It is linked to stimulation of: A. Acetylcholine synthesisB. (1-adrenoceptorsC. beta 1- adrenoceptors D. (2- adrenoceptorsE. M-cholinoceptors

138. The 42 y.o. patient has been suffering from bronchial asthma and tachycardia for a long time. Administer the optimal drug for removing of bronchospasm:A. AdrenalineB. EphedrineC. OrciprenalineD. IsadrinumE. Salbutamol

139. A patient has anaphylactic shock. Name a drug of choice.A. Mesatone (Phenylephrene)B. AdrenalineC. EphedrineD. NoradrenalineE. Fenoterol

140. The drug from group of beta-adrenoblockers has been prescribed to 47 y.o. patient for the treatment of ischemic heart disease. When will beta-adrenoblockers be absolutely contraindicated?A. Arterial hypertensionB. Bronchial asthmaC. Hypertrophic cardiomyopathyD. ThyrotoxicosisE. History of myocardial infarction

Page 29: Pharmacology MCQS for 3rd yr ZSMU

141. To the patient with an acute rhinitis 5% solution of ephedrine has been prescribed. However, the patient instilled the drug in the nose each 20-30 min. Because of development tachyphylaxis, effect of the drug sharply decreased. What mechanism of underlies development of this phenomenon?A. Depletion of mediator depot in presynaptic endingsB. Activation of adrenoceptorsC. Blockade of adrenoceptorsD. Material cumulationE. Induction of microsomal enzymes by liver

142. An ophthalmologist with diagnostic purpose (dilation of the pupil for observation of eye bottom) has used 1% solution of mesatone (phenylephrene). What mechanism determines mydriasis caused by the agent?A. Activation of (1-adrenoceptorsB. Activation of (2-adrenoceptorsC. Blockade of (1- adrenoceptorsD. Blockade of (1- adrenoceptorsE. Activation of M-cholinoceptors

143. A 64 y.o. woman with toxic goitre complains on permanent palpitation. What agent should be administered for correction of the heart rate?A. IsadrineB. SalbutamolC. PentamineD. Propranolol (Anaprilinum)E. Fenoterol

144. A patient has marked dropping of AP during surgical operation with hygronium administration. Representatives of what pharmacological group could normalize AP in given situation?A. N-cholinomimeticsB beta 1 -adrenomimeticsC. (1- adrenoblockersD. (2- adrenomimeticsE. M-cholinomimetics

145. After tooth extraction patient has developed local bleeding. A dentist applied sponge with a medicine and bleeding subsided. What medicine did the dentist use?A. IsadrineB. SalbutamolC. Adrenaline

Page 30: Pharmacology MCQS for 3rd yr ZSMU

D. PrazosinE. Timolol

146. A patient with chronic bronchitis has been taking with ephedrine for a long time. What is the drug’s action mechanism?A. Stimulation of noradrenaline release into synaptic cleftB. Blockade of noradrenaline release into synaptic cleftC. Stimulation of (-adrenoceptorsD. Blockade of (-adrenoceptorsE. Direct spasmolytic action

147. Indicate the state, which requires ephedrine introduction:A. TachycardiaB. InsomniaC. Caffeine poisoningD. Arterial hypotensionE. Arterial hypertension

148. A patient with obstructive bronchitis has been taking ephedrine for a long time without doctor’s control. What side effect can be observed in the patient?A. ApathyB. SleepinessC. Excitation of the CNSD. BradycardiaE. Hypotension

149. Collapse developed in a patient due to decrease of peripheral vessels tone. Which agent is useful in this situation?A. MetoprololB. Mesaton (Phenylephrine)C. XylomethazolineD. Proserine (Neostigmine)E. Prasosine

150. A patient with bronchial asthma had been taking orally an agent, which caused insomnia, headache, increased AP. What medicine can cause such complication?A. EphedrineB. IsadrinumC. PrasosineD. AdrenalineE. Dopamine

Page 31: Pharmacology MCQS for 3rd yr ZSMU

151. The patient suffering from idiopathic hypertension had been administered the complex treatment. Later he began to complain of pains in the epigastric region, diarrhea, heartburn. Which agent could provoke the listed adverse effects in the patient?A. Sympatholytics (Reserpine)B. Ganglioblockers (Pentamine)C. (-adrenoblockers (Anapriline))D. (1- adrenoblockers (Prasosine)E. Spasmolytics (Papaverine)

152. A patient who had been suffering from arterial hypertension was treated with an agent which mechanism of action is connected with exhaustion of norepinephrine content in sympathetic nerve ending. Indicate this agent:A. ClophelinumB. AnaprilineC. PrasosineD. MetoprololE. Reserpine

153. A patient who has been suffering from severe form of arterial hypertension after examination was diagnosed pheochromocytoma (tumor of adrenal medulla which is accompanied by increased synthesis of epinephrine). Indicate the drug group to treat patient before surgical treatment:A. alpha-adrenoblockersB. Alpha-adrenomimeticksC. GanglioblockersD. SympathomimeticsE. beta-adrenoblockers

154. A 58-year-old patient with arthritis during treatment with aspirin complains on nausea, distress in epigastrium. The doctor has cancelled aspirin and prescribed a NSAIA that is a selective cyclooxygenase-II inhibitor. Indicate the required agent for the patient:A. NaloxoneB. Paracetamol (Acetaminophen)C. MeloxicamD. Diclofenac-natrium (Voltaren)E. Tramadole

155. Which of the following drugs interrupt the cyclooxygenase pathway of eicosanoid synthesis by nonselective inhibiting both cyclooxygenase-1 and -2 (COX-1 and -2)?

Page 32: Pharmacology MCQS for 3rd yr ZSMU

A.MeloxicamB.CelecoxibC.NimesulideD.Diclofenac-natrium E.Allopurinol

156. In addition to providing symptomatic, supportive care, which of the following drugs would be a helpful adjunct to manage severe aspirin poisoning?A. AcetaminophenB. AcetylscysteineC. DiazepamD. Sodium bicarbonate E. Unitiolum

157. Aspirin causes anti-inflammatory action in result of depression of cyclooxygenase activity. Level of what biologically active substances will drop? A. Biogenic aminesB. CatecholaminesC. LeukotriensD. IodthyroninesE. Prostaglandins

158. At Aspirin overdose, method of urine alkalization is used. This method is directed to: A. Stimulate a secretion of the acid in proximal tubulesB. Neutralize acid in bloodC. Stimulate reabsorption of the acid in tubules D. Suppress a secretion of the acid in proximal tubulesE. Suppress reabsorption of the acid in renal tubules

159. Pains in the back developed after the lifting of heavy loads. Lumbosacral radiculitis was diagnosed. It is known from the anamnesis that the patient was suffering from ulcer of the duodenal bulb for a long period of time. Make the most rational choice of nonopioid analgesics:A. ButadionB. Ortophenum (diclofenac sodium)C. IndomethacinD. Meloxicam E. Ibuprofen

160. Gum bleeding arose in the patient after extraction of the tooth. From anamnesis it was revealed that the patient suffered from

Page 33: Pharmacology MCQS for 3rd yr ZSMU

rheumatic arthritis, and was treated with the anti-inflammatory agent acetyl-salicylic acid (aspirin). Indicate the reason of arisen bleeding.A. Suppression of synthesis of uric acidB. Promotion of thrombolysis C. Inhibition of hemopoesisD. Decreasing of blood coagulationE. Suppression of thromboxane synthesis

161. Utilization of arachidonic acid via cyclooxygenase pathway results in formation of some bioactive substances. Name them: A. Biogenic aminsB. Insulin-like growth factorsC. ThyroxineD. Prostaglandins E. Somatomedins

162. A patient with rheumatoid arthritis was prescribed a NSAID – Diclofenac sodium. After a period of time the patient experienced aggravation of a concomitant disease that forced to withdraw the drug. What concomitant disease could lead to the drug’s withdrawal? A. Diabetes mellitusB. Ischaemic heart diseaseC. Peptic ulcer of a stomachD. Bronchial asthmaE. Hypertensive disease

163. Aspirin causes anti-inflammatory action in result of depression of the cyclo-oxygenase activity. Level of what biologically active substance will drop? A. Biogenic aminesB. ProstaglandinsC. IodthyroninesD. LeukotrienesE. Catecholamines

164. A 60 y.o. patient with rheumatoid polyarthritis has been taking indometacin for a long time. What is the mechanism of action of this drug?A. Blockade of lipoxygenaseB. Blockade of acetylcholinesteraseC. Blockade of cyclooxygenaseD. Blockade phosphodiesteraseE. Blockade of phospholipase

165. A patient with severe, acute trauma pain requires analgesia. The physician orders morphine. Which of the following coexisting

Page 34: Pharmacology MCQS for 3rd yr ZSMU

conditions would pose the greatest risk from morphine’s use in this case?Acute pulmonary edemaHypertensionClosed head injury Opioid abuseRecent myocardial infarction

166. At postoperative period a patient has been receiving Promedol for a long time. After a cancellation of the drug the patient developed serious mental, neurological and somatic disturbances. Define this symptom-complex. A. IdiosyncrasyB. Rebound syndromeC. TachyphylaxisD. ‘Stealing’ syndromeE. Abstinent syndrome

167. To reduce the pain at traumatic shock Morphine hydrochloride has been given. What is the principal mechanism of analgesic action of it? RBlock of peripheral sensation receptorsInhibition of prostaglandins synthesisBlockade of opioid receptorsStimulation of opioid receptorsBlockade of adrenoreceptors

168. A synthetic analgesic, which increases myometrium contraction and relax the neck of uterus is administered to a woman in labor. Indicate this agent.A. OmnoponumB. Promedol (Trimeperidine) C. MorphineD. FentanylE. Fenoterol

169. A patient has signs of acute poisoning with morphine: sharp miosis, loss of consciousness, decrease of BP and Cheyne-Stokes respiration. Administer the pharmacological antagonist:A. NalorphineB. PromedolumC. PentazocinumD. Naloxone E. Tramadol

Page 35: Pharmacology MCQS for 3rd yr ZSMU

170. These biological active substances in the CNS have analgesic activity and are similar to morphine. Determine this active substance.A. OpiatesB. GABAC. Beta-endorphin D. CyclooxygenaseE. Dopamine

171. The patient was prescribed a narcotic analgesic known as a derivative of cyclogexanol, agonist-antagonist of opioid receptors that is weaker than morphine. What is the agent?A. NaltrexoneB. FentanylC. Tramadol hydrochloride D. Codeine phosphateE. Naloxone

172. Why morphine is undesirable to use in patients with cardio cerebral trauma?A. Provokes cardiac arrhythmiasB. Reduces blood pressureC. Increases intracranial pressureD. Causes psychological and physiological dependenceE. Suppresses respiration

173. At admission department it has been delivered a 30-year-old man after car accident with hip fracture. He has decreased BP, thread-like pulse, local tenderness of the broken hip. What is necessary to give to the patient for traumatic shock prevention?A. TramadolB. NaltrexoneC. Fentanyl D. NaloxoneE. Aspirin

174. The patient has receiving morphine for a long time because of breast cancer. What does not belong to the pharmacological effects of morphine?ConstipationInhibition of respiratory centerSpasm of Oddi sphincterDecrease of rate of respirationMydriasis

Page 36: Pharmacology MCQS for 3rd yr ZSMU

175. Examination of a patient revealed extremely myotic pupils, sleepiness, infrequent Chain-Stoke’s respiration, urinary retention, slowing-down of heart rate, and enhancement of spinal reflexes. What substance caused the poisoning?A. Morphine B. AtropineC. BarbitalD. CaffeineE. Phosphacole

176. A 52-yars –old patient who had been suffering from urolithiasis was delivered to the emergency department with renal colic. A doctor administered atropine together with opioid analgesic with spasmolytic activity to prevent development of the pain shock. Choose this drug.A. TramadolB. Promedol (Trimeperidine) C. Triftazinum (trifluoperazine)D. VasopressinE. Sulpiride

177. Morphine has been administered to reduce the pain at traumatic shock. What is the analgesic action mechanism of the drug?A. Blockade of Na+ channelsB. Blockade of peripheral sensation receptorsC. Stimulation of opioid receptorsD. Inhibition of cyclooxygenaseE. Inhibition of prostaglandin synthesis

178. A 42 year-old man who has been injured in a car accident is brought to the emergency room. His blood alcohol level on admission is 250 mg/dL. Hospital records show a prior hospitalization for alcohol related seizures. His wife confirms that he has been drinking heavily for 3 weeks.What treatment should be provided to the patient if he goes into withdrawal?Diazepam PhenobarbitalPentobarbitalDiphenin (Phenytoin)None

179. A patient of 17 years addressed a doctor with complaints of insomnia manifested in hard falling asleep that led to fatigue, weakness, difficulty of learning. The clinical examination revealed the following: irritability, emotional instability, pulse and BP alteration. The doctor determined that insomnia was associated with

Page 37: Pharmacology MCQS for 3rd yr ZSMU

neurosis-like state and vegetovascular distonia. Choose the most rational agent for correction of this condition.A. DroperidolB. Nitrazepam C. Aminazine (Chlorpromazine)D. PhenobarbitalE. Galoperidolum

180. A patient has been taking a mixture prescribed by neuropathologist for neurasthenia for a week. The patient feels better, but has developed conjunctivitis, rash, inertia, decrease of memory. She is diagnosed bromism. What should be prescribed to decrease the symptoms?NaloxoneAtropinPananginNatrium Chloride Pilocarpine

181. Choose drug combination producing neuroleptanalgesia.A. Haloperidole+ParacetamoleB. Droperidole+NaloxoneC. Diclofenac-natrium+HaloperidoleD. Droperidole + Fentanylum (Innovar) E. Aminazine+ Nimesulide

182. In a psychiatric clinic during psychosis treatment the symptoms of medicinal Parkinsonism had appeared. What drug has been used at the treatment of psychosis?A. Lithium carbonateB. ClozapineC. DiazepamD. Aminazine (Chlorpromazine) E. Sulpiride (Eglonil)

183. A patient of 40 years is delivered in psychiatric clinic in a state of exaltation, aggression, delirium. What drug should be entered to the patient? A. ReserpineB. Aminazine (Chlorpromazine) C. DiazepamD. Diclofenac sodiumE. Sodium oxybutiras

Page 38: Pharmacology MCQS for 3rd yr ZSMU

184. A patient, getting treatment for neurosis with Sibason (Diazepam), is complaining on a toothache. A doctor prescribed an analgesic in the dose less than therapeutic average dose. What phenomenon doctor took in consideration while decreasing agent’s dose?A. SummationB. CumulationC. ToleranceD. Additive effectE. Potentiation

185. A woman with active life-style addressed to outpatient department complaining on bad mood, migraine, emotional liability, and chest pain. What is wise to administer for neurosis treatment taking into account that she spends a lot of time at work? A. Mezapam B. PhenazepamC. Triftazine D. PhenobarbitalE. Aminazine (Chlorpromazine)

186. A patient had been suffering from schizophrenia accompanied by arterial hypertension. A doctor administered neuroleptic possessing expressed hypotensive activity. Indicate this drug.A. HaloperidolB. RisperidoneC. Aminazine (Chlorpromazine) D. DiazepamTriftazinum (trifluoperazine)

187. A patient took a drug from the group of Benzodiazepines for the treatment of neurosis for a long period of time. One day the patient felt sleepiness, weakness, and decrease of memory, concentration and reactions, headache, nausea. The patient was addressed to the polyclinic and a doctor administered an injection of an antagonist of the Benzodiazepines. Determine this drug.A. BemegrideB. NaltrexoneC. NaloxoneD. FlumazenilE. Cordiamin

188. What group of drugs is used for the treatment and prevention of manias?A. AntidepressantsB. SedativesC. Neuroleptics

Page 39: Pharmacology MCQS for 3rd yr ZSMU

D. TranquilizersE. Lithium

189. Long application of the neuroleptics majority of which invokes and enhances symptoms of Parkinsonism, is necessary for the patient with pre-existing slight parkinsonism syndrome. What neuroleptic is least dangerous in that case?A. Aminazine (Chlorpromazine)B. DroperidoleC. TriftazineD. Clozapine (Leponex)E. Haloperidol

190. Aminazine was administered to a patient with schizophrenia. Which pharmacodynemic effect of aminazine is the basic for this patient?A. AnxiolyticB. Antipsychotic C. MyorelaxantD. HypotensiveE. Hypothermic

191. A patient of 28 years in a state of exaltation, aggression, delirium is delivered to psychiatric clinic. What drug should be administered to the patient?A. DiazepamB. Sodium bromideC. Tincture of ValerianD. Aminazine (Chlorpromazine) E. Lorazepam

192. For performing an operative measure, it is necessary to use a method of general hypothermia. At application of which agent in combination with physical refrigerating the expressed hypothermia is observed?A. DiazepamB. Sodium bromideC. Tincture of ValerianD. Aminazine (Chlorpromazine) E. Lorazepam

193. Glutamate decarboxylation results in formation of inhibitory transmitter in CNS. Name it: A. SerotoninB. GABA C. Glutathione

Page 40: Pharmacology MCQS for 3rd yr ZSMU

D. HistamineE. Asparagines

194. A 35 y.o. patient with schizophrenia complains on discoordination of movement, tremor of hands, drowsiness. The patient has been taking psychotropic agent for a long time. Which agents are most likely to cause these disturbances?A. TranquilizersB. Atypical neurolepticsC. Typical neurolepticsD. AntidepressantsE. Psychostimulants

195. Phenazepam has been prescribed to a patient with hyper excitability, irritability, tearfulness, sleeplessness. What is the drug action mechanism?A. Blockade of GABA-receptorsB. Stimulation of benzodiazepine receptorsC. Blockade of M-cholinoceptorsD. Blockade of phosphodiesteraseE. Stimulation of dopamine receptors

196. For the treatment of delirium and hallucination Aminazine (Chlorpromazine) has been prescribed. What is its antipsychotic action mechanism?A. Blockade of reuptake of catecholaminesB. Stimulation of adrenergic and dopaminergic processes in the CNSC. Inhibition of adrenergic and dopaminergic processes in the CNSD. Blockade of M-cholinoceptorsE. Stimulation of M-cholinoceptors

197. A doctor administered Aminazinum (chlorpromazine) to the patient suffering from schizophrenia to eliminate delirium, hallucinations, to decrease aggression and psychomotor excitement. What is the mechanism of antipsychotic action of Aminazinum?A. Excitation of M-cholinoreceptors in the CNSB. Stimulation of opioid reeptorsC. Blockade of D2-dopamine receptors in the the CNSD. Excitement of adrenoreceptors and dopamine receptors in the CNSE. Inhibition of MAO

198. A 32 y.o. woman was addressed to a doctor with complaints of temper, tiredness, insomnia, internal tension. The doctor had

Page 41: Pharmacology MCQS for 3rd yr ZSMU

diagnosed neurosis and administered tranqilizer (diazepam). Which of the effects of this agent is more important in this situation?A. AntipsychoticB. AnxiolyticC. AnticonvulsiveD. MyorelaxantE. Antiemetic

199. A dentist introduced sibazonum (diazepam) to a 47 y.o. woman before extraction of tooth. Indicate the anxiolytic action mechanism of the drug:A. Stimulation of opioid receptorsB. Inhibition of dopamine receptors and adrenoceptorsC. Stimulation dopamine receptors and adrenoceptorsD. Agonist of benzodiazepine receptorsE. Inhibition of benzodiazepine receptors

200. A patient was addressed to a doctor with complaints of irritability, insomnia, fatigue. A doctor administered a sedative drug to him. In a week the patient began to complain of cough, sleepiness, decrease of memory, phenomena of rhinitis, conjunctivitis, dermatitis. Which group of the drugs was administered by the doctor?A. NeurolepticsB. TranquilizersC. LithiumD. BromidesE. Non-steroidal anti-inflammatory agents

201. Drug of choice for psychomotor epilepsy is:A. Valproic acidB. Carbamazepine C. EthosuximideD. BarbiturateE. Diphenin

202. Which anti-epileptic acts by augmentation of release of inhibitory transmitter GABA by inhibiting its degradation (by GABA-transaminase) as well as probably by increasing its synthesis?A. Valproic acid B. Carbamazepine C. Diphenin (Phenytoin)D. EthosuximideE. Barbiturates

Page 42: Pharmacology MCQS for 3rd yr ZSMU

203. Which anti-epileptic acts by prolongation of Na+ channels inactivation as well as by inhibiting kindling? The drug also has antidiuretic action, probably by enhancing ADH action on renal tubules.A. Carbamazepine B. Phenobarbital C. DroperidoleD. Valproate sodiumE. Ethosuximide

204. A major problem that must be faced when administering anticonvulsants with many other medications (including other antiepileptic drugs) involves drug interactions due to altered metabolism. Which of the following drugs is likely to cause excessive or toxic effects from some other drugs by inhibiting metabolism? A. Ethosuximide B. CarbamazepineC. PhenobarbitalD. Phenytoin E. Valproic acid

205. A man develops akathisia,a Parkinson-like syndrome, galactorrhea, and amenorrhea, during the therapy. Which of the following mechanisms is present?A.Blockade of muscarinic receptorsB. Blockade of dopamine receptors C. Blockade of +-adrenergic receptorsD. Stimulation of dopamine receptorsE. Supersensitivity of dopamine receptors

206. The doctor has prescribed Cyclodolum for Parkinson disease treatment. What is the mechanism of anti parkinsonian action of the drug?A. M cholinomimetic actionB. M cholinoblocker action C. Stimulation of dopamine receptorsD. Blockade of dopamine receptorsE. (-Adrenoblocker action

207. A patient of 58 year old has been taken Phenobarbital in connection with insomnia. After a cancellation of the drug the patient again has difficulty in falling asleep, frequently wakes up at night; dreams is accompanied by dreadful dreams. What is the reason of the given undesirable effect of Phenobarbital?A. DependenceB. IdiosyncrasyC. Tachyphylaxis

Page 43: Pharmacology MCQS for 3rd yr ZSMU

D. Rebound syndrome E. Tolerance

208. A patent defied doctor’s instructions and continued to use Phenobarbital over 3 weeks. Meanwhile, he was forced to increase the drug’s dose. How to explain falling of Phenobarbital efficacy?A. Activation of lipolysisB. Inhibition of monooxygenase systems of a liverC. Induction of monooxygenase systems of a liver D. Inhibition of lipolysisE. Induction of glycolysis

209. A 56 y.o. man with insomnia was given Phenobarbital. The sleep was normalized. However, after 2 weeks the effect of Phenobarbital fell down. What reason caused decreasing of hypnotic action of Phenobarbital?A. Its poor solubilityB. Development of tolerance C. Its poor absorption in stomachD. Its fixation in lipidsE. Development of tachyphylaxis

210. A patient with epilepsy was recommended a medicine known as a derivative of dipropilacetic acid that inhibits GABA-transferase and increases GABA level in the brain; inhibits excitability and seizure capability of motor zone of the CNS. It can be used for all types of epilepsy. Name this medicine:A. ReserpineB. Diazepam (Seduxen)C. Diphenin D. Sodium bromideE. Sodium valproate

211. Prescribe a drug that belongs to modern hypnotics to the patient with insomnia. It is an imidazopyridine derivative; activates benzodiazepine receptors in the CNS; does not alter either sleep structure or activity of liver enzymes, does not cause dependence.A. DroperidolB. NitrazepamC. Zolpidem D. Chloral hydrateE. Flumazenil

212. What phenomena may be developed in long taking of barbiturates? Drug tolerance

Page 44: Pharmacology MCQS for 3rd yr ZSMU

Drug dependence Extrapyramidal syndrome SensibilizationInhibition of enzymes of liver

213. The patient has appealed to the dentist with complaints of hypertrophy and pain in the gums. After examination of the oral cavity the doctor has diagnosed hyper plastic gingivitis. It has been established from the anamnesis, that the patient has been taking for a long time an antiepileptic agent. Specify this drug.A. Diphenin (Phenytoin) B. Hexamidinum C. PhenobarbitalD. CarbamazepineE. Trimethinum (trimethadione)

214. A patient has trauma-caused brain edema and hypoxic cramps. What agent is rational to use for abortion of the cramps?A. CordiaminumB. Anaprilinum (Propranolol)C. BemegrideD. Sodium oxybutyras E. Piracetam

215. A 57 y.o. man in a coma has been delivered to hospital. It is known from his anamnesis that he suffered from insomnia. At inspection the following is revealed: respiratory depression, fallen AP, progressing cardiac weakness, decrease of body to, inhibition of tendon reflexes. What drug could cause poisoning?A. FluoxetineB. Phenobarbital C. Tincture of ValerianD. LevodopaE. Sodium bromide

216. A woman after the experienced nervous shock badly sleeps. What agent should be administered?A. FluoxetineB. Phenobarbital C. NitrazepamD. Aminazine (Chlorpromazine)E. Sodium bromide

Page 45: Pharmacology MCQS for 3rd yr ZSMU

217. Diphenin (Phenytoin) has been administered to a patient for epilepsy with tonic-clonic attacks. What is the action mechanism of the drug?A. Activation of GABA systemB. Blockade of Ca2+ channelsC. Stimulation of dopamine receptorsD. Blockade of dopamine receptorsE. Blockade of Na+ channels

218. A 76 y.o. patient has appealed to the doctor with complaints of superficial short-term sleep with often awakenings caused by sense of internal tension, anxiety, fear. Senile sleeplessness was diagnosed. Make a rational choice of a hypnotic in the given situation:A. Phenobarbital B. Chloral hydrateC. NitrazepamD. LamotrigineE. Carbamazepine

219. A patient with convulsions was delivered to the hospital where status epilepticus was diagnosed. Indicate the first choice drug to treat the patient:A. CarbamazepineB. DiazepamC. PhenobarbitalD. LevodopaE. Chloral hydrate

220. An attack of generalized tonic-clonic convulsions accompanied by loss of consciousness and general suppression of the CNS developed in a patient after trauma. Which agent should be administered to this patient? A. Cyclodol (Trihexyphenidyl)B. Midantan (Amantadine)C. LevodopaD. Soidium oxybutiras E. Phenobarbital

221. Symptoms of poisoning by derivative of barbituric acid are revealed in a patient. What drug can increase excretion of barbiturates from the organism?A. Natrium chlorideB. Natrium hydrocarbonate C. Kalium chloride

Page 46: Pharmacology MCQS for 3rd yr ZSMU

D. Magnesium oxideE. Magnesium sulfate

222. Determine the hypnotic drug, which doesn’t influence the structure of sleep.A. BarbitalB. PhenobarbitalC. Nitrazepam D. Aethaminalum-natrium (pentobarbital)E. Bromisoval

223. MAO inhibitors are contraindicated with all of the following EXCEPT:A. Indirect adrenergic agents, such as ephedrineB. Tricyclic antidepressantC. Paracetamol D. Beer and cheese E. Dopamine

224. The patient addressed to a doctor with complaints of bad mood, that was accompanied by expressed sedative action, feeling of fear, anxiety. The clinical examination revealed psychical depression. What drug should be administered to this patient?Caffeine Sulfocamphocaine Pyracetam Amitriptyline Droperidol

225. A doctor administered amitriptyline to a patient with to a patient endogenous depression. Explain the mechanism of action of this drug.A. Inhibition of the neuronal reuptake of noradrenalineB. Inhibition of the neuronal reuptake of serotoninC. MAO inhibitorD. Increase of release of noradrenaline and serotoninE. Inhibition of the neuronal reuptake of noradrenaline and serotonin

226. A man was addressed to the psychiatrist with complaints and desperation, tendency to suicide. Determine the group of drugs for the treatment of this patient?A. Antidepressants B. SedativesC. Neuroleptics

Page 47: Pharmacology MCQS for 3rd yr ZSMU

D. TranquilizersE. Lithium

227. This group of drug includes psychotropic agents of plant origin. These drugs are used in asthenic states after severe infectious diseases. They increase general vital tonicity of the organism and its resistance to infectious diseases. These drugs are used in the form of tinctures or liquid extracts. Choose this group of drug.A.SedativeB.Adaptogens C. PsychostimulantsD.AntidepressantsE.Nootrop agents

228. This drug has stimulating action promoting synthesis of proteins and ATP, increases the capacity for physical and psychical work. It is used for a long time and it shouldn’t be taken before sleep.Tincture of Ginseng Tincture of ValerianBarboval NialamideNatrium bromide

229. Analeptic of reflective type from the N-cholinomimetics group was given to the patient for restoration of breathing after poisoning with carbon monoxide. What agent was administered to the patient?Adrenaline hydrochlorideLobeline MesatonNaloxoneCaffeine

230. Indicate the mechanism of action of Caffeine.A. Blockade of Monoaminooxidase (MAO)B. Inhibition of the neuronal capture of NoradrenalineC. Inhibition of Phosphodiesterase (PDE)D. Blockade of Adenosine receptors.E. Blockade of adenosine receptor and inhibition of phosphodiesterase

231. Indicate the condition for the use of Piracetam:A. Significant atherosclerosisB. Mental insufficiency C. ConvulsionsD. Hypertensive crisisE. Convulsions

Page 48: Pharmacology MCQS for 3rd yr ZSMU

232. Indicate the analeptic with the mixed type of action:A. Cordiamin B. CaffeineC. Lobeline hydrochlorideD. BemegrideE. Aethimizolum

233. Indicate the analeptic with the direct type of action:A. Cordiamin B. CytitonC. Lobeline D. Bemegride E. Sulfocamphocaine

234. During the operation under general anesthesia the patient’s respiration was inhibited stimulant should be used in this stimulant without pausing the general anesthesia.A. Aethymisole B. CaffeineC. BemegrideD. CytitonE. Lobeline hydrochloride

235. Indicate the drug which possesses analeptic and psycho stimulant activity.A. Caffeine B. BemegrideC. AethimizolumD. Cordiaminum (nikethamide)E. Strychnine

236. Specify application for use of Bemegride:A. ParkinsonismB. Hypertensive crisesC. Overdose of general anesthetics D. Brain hypoxiaE. Tachyarrythmias

237. The patient was admitted to the neurology department because of complaints of decrease of memory, mental and work capacity, sleeplessness and vertigo. His symptoms were connected to a brain concussion, which took place 2 years ago as a result of an automobile accident. What drug should be indicated to improve his condition?

Page 49: Pharmacology MCQS for 3rd yr ZSMU

A. Pyracetam (Nootropil) B. Natrii oxybutiras (oxybyte sodium)C. SydnocarbD. CaffeineE Cordiamin (nikethamide)

238. A 28 y.o. patient was delivered to the intensive care unit with the diagnosis of poisoning with barbiturates of mild degree. Administer a stimulant of respiration, which will be pertinent in this case:A. InstenonB. BemegrideC. SydnocarbD. PiracetamE. Diazepam

239. Thiopental natrium was introduced to the patient for initial general anesthesia that caused the arterial hypotension and inhibition of breathing. Select a drug used for restoration of breathing and blood circulation.A. LobelineB. DipiroximC. NaloxoneD. CytitonE. Bemegride

240. The child was born in a state of asphyxia. What drug is necessary to administer to the newborn for stimulation of respiration? A.Proserine (Neostigmine) B.Naloxone C.Prazosin D.Atropine E.Etimizol

241. A patient with major depression has been prescribed Nialamide. The doctor has warned the patient about necessity to exclude from diet during the treatment:

A.Cheese B.Apples C.Potato D.Cabbages E.Pears

Page 50: Pharmacology MCQS for 3rd yr ZSMU

242. To the patient with a bipolar manic-depressive illness in a stage of depression, which complained of feeling of alarm, fear, the antidepressant with concomitant sedative effect has been prescribed. What was a drug?A. Nialamide B.Imizine (Imipramine) C.Sydnocarb D.Amitriptyline E.Levodopa

243. A patient after cranial trauma was delivered to the neurological department. Progressive increase of neurological symptoms allowed making the diagnosis of brain contusion, which became the reason of increasing edema of the brain followed by hypoxia of its structures.Administer to the patient a general anesthetic, which exerts beneficial influence in brain hypoxia:A. PhthorotanB. IsofluranC. Propanidide (Sombrevine)D. KetamineE. Natrii oxybutiras (GOBA)

244. In severe alcohol intoxication, the blood ethanol exceeds:A. 0.5 g/L B. 1 g/L C. 2 g/L D. 3 g/L E. 4 g/L

245. The same dose of alcohol invokes a different degree of alcohol intoxication in different people. It is linked with:A. High plasma level of beta-lipoproteins B. Inhibition of insulin synthesisC. Genetically determined insufficiency of alcohol dehydrogenase synthesisD. Inhibition of hepatic microsomal enzymesE. Low activity of pseudocholinesterase (butyrylcholinesterase)

246. To anaesthetize the manipulation related to a burned surface treatment, a patient was IV injected a medication for short-acting narcosis. 1 minute later the patient being under anesthesia had an increased BP, tachycardia, increased tone of skeletal muscles; reflexes remained. After awakening the patient had disorientation and visual hallucinations. What medication was the patient injected?A. Nitrous oxideB Thiopental sodiumC Diethyl ether

Page 51: Pharmacology MCQS for 3rd yr ZSMU

D SombrevinE Ketamine

247. A patient is 25 years old with a poly trauma, unconscious, the AP is 95/70 mm Hg, pulse is 75 beats per minute, respiration superficial with frequency 30 in one min. What agent should be selected for a narcosis (general anesthesia)?A. Ketamine B. Thiopental sodiumC. Diethyl etherD. SombrevinE. Fluorotane (Halothane)

248. Thiopental sodium was IV administered to a patient for an introduction narcosis and then the laryngospasm and hypersalivation have developed. Administration of what drug could prevent undesirable effects?A. Nitrazepam B. CarbamazepineC. AlloximeD. AtropineE. Flumazenil

249. With the purpose of warming after cooling, the patient used solution of ethanol.How does ethanol influence on thermoregulation?A. Increases heat release B. Decreases heat releaseC.Increase heat productionD. Decreases heat productionE. Decreases heat production and heat release

250. The doctor has prescribed teturam (disulfiram) for the treatment of the patient with alcoholism. What is the mechanism of action of the given drug?A. Activation of vomiting CenterB. Augmentation of AcetylcholinesteraseC. Inhibition of AcetylcholinesteraseD. Augmentation of aldehyde dehydrogenase activityE. Inhibition of Aldehyde Dehydrogenase

251. A 15-years-old boy was admitted to the hospital with polytrauma in unconscious state, BP 95/70 mm Hg, heart rate 72 per minute, superficial breathing (respiration rate is 30 per minute). Choose the drug for general anaesthesia.A. Propanidid

Page 52: Pharmacology MCQS for 3rd yr ZSMU

B. Hexenalum (hexobarbital)C. Ketamine D. Thiopental natriumE. Phthorotanum (halothane)

252. General anesthetic with ultra short action (5 minutes) was introduced to a patient intravenously for analgesia of biopsy. It caused muscular twitching, insignificant decreased of blood pressure, stop of breathing for short period of time. Determine this drug.A. Ketamine B. Natrii oxybutiras C. PentazocinumD. Propanidid E. Phthorotanum (halothane)

253. Propanidid is used very often in dentist practice. Determine the contraindication for use of this drug.A. Reposition of jawbone fragmentsB. Shock C. Reposition of dislocations of jawD. Taking out the stitchesE. Dissection of pulp cavity

254. The patient used solution of ethyl alcohol for warming after super cooling. How does ethyl alcohol influence upon thermoregulation?A. Decreases heat production and heat emissionB. Decreases heat emissionC. Increases heat productionD. Decreases heat productionE. Increases heat emission

255. Expressed bradycardia, reducing of AP appeared during phthorotanum anesthesia. What drug is contraindicated to normalize the heart rate for continuation of the operation?A. Proserpine (neostigmine)B. AtropineC. Mesatonum (Phenylephrine)D. CaffeineE. Adrenaline (epinephrine)

256. A patient suffers from chronic alcoholism with the following symptoms: pain in arms and legs, impairment of skin sensitivity, muscle weakness, edemas and increased amount of private. Which vitamin drug should be prescribed to the patient?A. Ergocalcipherol

Page 53: Pharmacology MCQS for 3rd yr ZSMU

B. Thiamine C. RetinalD. RutinE. Vikasolum (Menadione)

257. A 35 y.o. patient who often consumes alcohol was treated with diuretics. There appeared serious muscle and heart weakness, vomiting, diarrhea. AP – 100/60 mm Hg, depression. This condition is caused by intensified excretion with of: A. CalciumB. PotassiumC. ChlorineD. PhosphatesE. Sodium

258. A patient, who has been treating with Clophelinum (Clonidine) due to arterial hypertension, has taken an alcoholic drink that has caused the sharp inhibition of the CNS. What is the name of this interaction?A. Potentiation B. SummationC. CumulationD. IntoxicationE. Idiosyncratic reaction

259. A sudden acute decrease of arterial pressure arise in a patient who was being operated under general anesthesia. The doctor introduced epinephrine following by ventricle fibrillation. Indicate general anesthetic, which can most likely induce this complication:A. Nitric oxideB. PropanidideC. Phthorotanum (Halothane)D. KetamineE. Thiopental-sodium

260. A patient of 50 years old with complaints of heart palpitation, pain in the heart area, headache, and vertigo was delivered to a hospital. During the examination angina pectoris, disorders of cardiac rhythm (paroxysmal form of atrium fibrillation) and arterial hypertension (180/90 mm Hg) were revealed. Make the rational choice of the drug for this patient:A. Propranolol B. Epinephrine C. Enalapril D. Nifedipine E. Digoxin

Page 54: Pharmacology MCQS for 3rd yr ZSMU

261. During the ambulant tooth extraction a patient who had been suffering from chronic congestive heart failure showed the typical signs of acute heart failure: dyspnea, cyanosis, tachycardia, ps 96 per minute, BP 100/60 mm Hg. Indicate the drugs to be used to treat this patient.A. Sthrophanthine + VerospironeB. Lisinopril + FurosemideC. Anapriline + SulfocamphocaineD. Sthrophanthine + FurosemideE. Anapriline + Furosemide

262. A 35-y.o. male has undergone surgery for necrotic bowel. Despite having been treated with antibiotics, on the 5th postoperative day, he develops symptoms (fever, hypotension, tachycardia, declining urine output, confusion) consistent with septic shock. What hemodynamic support would be helpful?A. Fluid administrationB. Dobutamine infusionC. Fluid and dobutamine infusion D. Atropine administrationE. Antibiotic administration

263. The action of electric current on the excitable myocardial cell caused Depolarization of its membrane. Movement of what ions through the membrane caused depolarization?A. Ca2+B. K+C. Cl-D. Na+ E. HCO3

264. Treatment of Digitals toxicity can include all except:A. PotassiumB. Phenytoin C. LidocaineD. Dialysis E. Unitiolum

265. A 45 y.o. patient has been prescribed Digoxin to treat chronic heart failure. What is the mechanism of action of Digitoxin?A. Inhibition of Na+/K+- ATPaseB. Decreases intracellular Na+ concentrationC. Increases the intracellular level of ATPD. Stimulates production of cAMPE. Decreases release of Ca2+ from the sarcoplasmic reticulum

Page 55: Pharmacology MCQS for 3rd yr ZSMU

266. A patient with a cardiac arrhythmia is being treated for a long time with Amiodarone. This drug can cause biochemical changes and clinical signs and symptoms that resemble those associate with which of the following disease?A. Diabetes mellitusB. Addisonian crisisC. HypothyroidismD. Diabetes insipidusE. Cushing’s syndrome

267. Which antiarrhythmic drug has relatively few electrophysiologic effects on normal myocardial tissues, but suppresses the arrhythmogenic properties of ischemic tissues?A. Digoxin B. PropranololC. LidocaineD. AmioadroneE. Phenitoin

268. Specify the mechanism of antiarrhythmical action of Amiodaronum (Cordaronum):A. Blockade of Na+ channels of the cardiomyocyte membranesB. Blockade of Ca2+ channels of the cardiomyocyte membranes C. Blockade of beta – adrenoceptors of the myocardiumD. Blockade of alfa – adrenoceptors of the myocardiumE. Blockade of K+ channels of the cardiomyocyte membranes

269. Antiarrhythmic with greatest repolarization phase is:A. QuinidineB. Amiodarone C. Lidocaine D. PropranololE. Verapamile

270. In terms of therapeutic usefulness, the most important pharmacologic action of digoxin in congestive heart failure is:A. The reduction of cardiac sizeB. The increase in ventricular contractile forceC. The slowing of heart rateD. The diuretic effectE. The increase in blood pressure

271. The patient with a cardiogenic shock has been delivered into an emergency room. Choose a drug of a first choice:

Page 56: Pharmacology MCQS for 3rd yr ZSMU

A. Dopamine B. Propranolol C. Cordiamine D. Adrenaline hydrochlorideE. Enalapril

272. The drug of choice in anaphylactic shock is:A. Dopamine B. Propranolol C. Cordiamine D. Adrenaline hydrochlorideE. Enalapril

273. The half life of digoxin is:A. 12 hrsB. 24 hrsC. 36 hrsD. 5 daysE. 2 weeks

274. In a clinic, the patient complained of unpleasant sensations in the heart region, and attacks of weakness and loss of consciousness. Inspection of the patient’s electrocardiogram had revealed the presence of II degree atrioventricular block. Specify a drug, which should be used in the situation:A. Isadrinum B. NovocainamideC. NitroglycerineD. Strophanthin E. Anaplilinum

275. A 50 y.o. patient with chronic cardiac insufficiency and tachyarrythmia was prescribed a cardiotonic drug. What drug was prescribed?A. DobutamineB. AmyodaroneC. DopamineD. Digoxin E. Mildronate

276. Digitoxin (1 tab tid) had been prescribed to a patient with chronic (congestive) heart failure. In 15 days, the signs of intoxication developed in the patient – bradyarrhythmia, disturbance of atrioventricular conduction, upset of colour vision. What could cause the specified phenomena?

Page 57: Pharmacology MCQS for 3rd yr ZSMU

A. Functional cumulationB. Material cumulation C. PotentiationD. Tolerance E. Summation

277. In homozygous mice with defect of the gene-coding frame of a beta1—adrenoreceptors, it has been probed action of an agent on inotropic function of heart. What agent will lose the activity in these conditions?A. DobutamineB. Mesaton (Phenylephrine)C. StrophanthinD. Caffeine E. Digoxin

278. A water-soluble cardiac glycoside has been prescribed to a patient. What concomitant pathology can promote a cumulation of water-soluble cardiac glycosides?A. Renal failure B. Liver failureC. Hyperacide gastritisD. Hypoacidic gastritis E. Hypothyroidusm

279. Indicate the mechanism of anti arrhythmic action of Quinidine sulphate.A. Blockade of Na+ channels of the cardiomyocyte membranes B. Blockade of Ca2+ channels of the cardiomyocyte myocardium C. Blockade of beta – adrenoceptors of the myocardiumD. Blockade of alfa – and beta – adrenoceptors of the myocardiumE. Blockade of M-cholinoceptors of the myocardium

280. Indicate the group of drugs, overdosage of which is accompanied by following signs: nausea, vomiting, diarrhea, infringement of heart activity (extrasystoles, delay of antrioventricular conductivity), headache, vision impairment (xanthopsia, diplopia).A. Ca2+ - channels blockersB. Organic nitratesC. Cardiac glycosides D. Beta - adrenoblockers E. Angiotensin converting enzyme inhibitors

Page 58: Pharmacology MCQS for 3rd yr ZSMU

281. A patient suffers from allergic reaction to iodine. Indicate an anti-arrhythmic agent, which is contraindicated to him.A. Amiodarone B. VerapamilC. Novocainamidum (Procainamide)D. Ornidum (bretylium) E. Quinidine sulfate

282. Indicate the drug and its group, which is used for treatment of atrioventricular blockade.A Cardiac glycoside DigitoxinB. Beta-adrenomimetic Isadrinum (isoprenaline) C. Sympatholytic Ornidum (bretylium)D. Beta –adrenoblocker Anaprilinum (Propranolol) E. Blocker of slow calcium channels Verapamil

283. Indicate the group of drugs used to treat total atrioventricular block:A. M-cholinoblockersB. Membrane-stabilizing agentsC. Local anestheticsD. Beta –adrenoblockerE. Potassium channels blockers

284. Symptoms of cardiac glycosides toxicity have been developed in a patient with chronic congestive heart failure. What agent should be administered to decrease adverse effects of cardiac glycosides?A. EthimisolB. DipiroximC. Potassium chlorideD. Atropine sulfateE. Novocainamid

285. Owing to a myocardial infarction, the patient had ventricular arrhythmia. The cardiac rhythm was normalized after introduction of an antiarrhythmic agent with local anesthetic activity. What drug was introduced?A. VerapamilB. Chinidinum (Quinidine)C. SotalolD. PropranopolE. Lidocaine

Page 59: Pharmacology MCQS for 3rd yr ZSMU

286. An atrioventricular blockade has been revealed in a 25y.o man during inspection. Name the agent used for treatment of atrioventricular blockade:A. AmiodaronB. Proserinum (Neostigmine)C. VerapamilD. Metoprolol E.Isadrinum

287. A patient has an attack of tachycardia. What type of membrane receptors of cardiomyocytes is wise to block for stopping of the attack?A. M-cholinoreceptorsB. Beta1 – adrenoceptors C. Beta2 – adrenoceptors D. Alfa- adrenoceptors E. M-and N- cholinoreceptors

288. For correction of the arrhythmias caused by poisoning with cardiac glycosides, the antiepileptic agent diphenin (phenytoin) can be used. What effect is typical for diphenin (phenytoin)?A Quickens repolarization and reduces the efficient refractory period B. Blocks beta- adrenoceptors C. Decreases contractility of myocardium D. Suppresses contractility of heart E. Prolongs Phase 3 repolarisation

289. The permeability of the irritable cell membrane has been increased for potassium ions during an experiment. What changes of membrane electric status can occur?A. DepolarizationB. Action potentialC. No changesD. Local responseE. Hyperpolarization

290. A patient with chronic cardiac insufficiency has been treated with cardiotonic drugs and a thiazide diuretic, but in spite of it there are still edemata and risk of ascites. What medication should be prescribed to amplify diuretic effect of the applied drugs?A. Spironolactone B. FurosemideC. ManitoleD. AmilirideE. Clopamide

Page 60: Pharmacology MCQS for 3rd yr ZSMU

291. Which of the following drugs is appropriate and generally regarded as most effective for relieving and preventing ischemic episodes in patients with variant angina?A. AspirinB. LovastatineC. PropranololD. DiltiazemE. Nitroglycerin

292. Nitrites used in management of angina have all the following action except:A. Increase total coronary blood flow B. Decrease cardiac O2 consumption C. Decrease preload and after load D. Redistribution of blood into subendocardial vessels E. Dilatation of cerebral vessels

293. In a patient with angina pectoris who is taking insulin to treat diabetes, which of the following drugs is to be used with extra caution and advice to the patient?A. AspirinB. VerapamilC. PropranololD. DiltiazemE. Nitroglycerin

294. Beta-blockers are used in all cases except:A. Variant angina B. Mild hypertensionC. GlaucomaD. ThyrotoxicosisE. Atrial arrhythmias

295. Which of the following drugs can relieve angina pectoris by decreasing myocardial work, but may precipitate congestive heart failure? A. PhentolamineB. SthrophanthineC. Propranolol D. AtropineE. Enalapril

Page 61: Pharmacology MCQS for 3rd yr ZSMU

296. The advantages of metoprolol over propranolol include all of the following except A. More likely to be effective in sinus tachycardia B. Less likely to provoke bronchoconstrictionC. More likely to be effective in sinus bradycardiaD. Less likely to produce temporary rise in peripheral resistance at the start of therapyE. Less likely to produce sudden rises in BP after physical exertion

297. All the following statements are true about Enalapril except A. A prodrugB. Used to treat hypertensionC. Used to treat CHFD. Inhibits ACEE. Blocks Angiotensin-1

298. Calcium channel blocker with maximum peripheral action is:A. Verapamil B. Nifedipine C. Diltiazem D. PapaverineE. Molsidomine

299. In a patient with myocardial infarction, if congestive cardiac failure develops, the dose of digoxin should be:A. Reduced B. Increased C. Unaltered D. None of the above

300. Drugs that may precipitate variant angina pectoris are:A. Beta blockers B. Nitrites C. Alpha blockersD. Calcium channel blockers E. Benzodiazepines

301. After tooth extraction the patient experienced persistence chest pain. In result of sublingual use of an antianginal agent the chest pain disappeared, but patient complained on headache and dizziness. What agent did the patient use?A Anaprilin (Propranolol) B. ValidolumC. Nitroglycerine (Glyceril trinitrate) D. Verapamil

Page 62: Pharmacology MCQS for 3rd yr ZSMU

E. Metoprolol

302. A patient had been suffering from stenocardia accompanied by arterial hypotension. Indicate the anti anginal drug, which does not cause decrease of blood pressure.A Pentoxyphilline B. AnaplilinumC. NitroglycerinumD. Amiodaronum E. Nifedipin

303. A 27 y.o. man with angina pectoris was admitted to a cardiologic department. A phosphodiesterase inhibitor was included to the complex therapy of the patient. Concentration of what substance will increase in the cardiac muscle?A ATPB. Cyclic-AMP C. ADPD. GMP E.AMP

304. A 32-years-old patient after intensive psychoemotional stress suddenly experienced acute pain in heart area irradiating into left arm. Nitroglycerin abolished the attack after 10 min. What pathogenic mechanism is the leading reason in pain development in this case?A Spasm of coronary vessels B. Increasing of oxygen demand of myocardiumC. Dilation of peripheral blood vesselsD. Compression of coronary vesselsE. Obstruction of coronary vessels

305. To the patient, suffering from stenocardia attacks, administration of long-acing nitrates is indicated. Find out this drug from listed.A. NitroglycerinB. Isosorbide mononitrate C. Sodium-nitroprussideD. VerapamilE. Phenihydine (Nifedipine)

306. A 48 y.o. patient with the diagnosis of ischaemic heart disease is getting medical treatment. Specify the antianginal agent that has benefits to improve endocardial circulation:A. Anaprilinum (Propranolol)B. Nitroglycerin

Page 63: Pharmacology MCQS for 3rd yr ZSMU

C. PrazosinD. PapaverineE. Euphylline (Theophylline)

307. A patient who had been treated with a vitamin agent for the prophylaxis of brain vessel constriction complained of unpleasant sensations: blushing of upper part of the body, vertigo, flushing of blood to the head. What agent exerts this effect?A. Nicotinic acidB. Thiamini bromideC. RiboflavinD. Tocopherol acetateE. Anaprilinum (Propranolol)

308. A 3 y.o. child is delivered to an admission department with signs of the nitrates poisoning such as cyanosis, dyspnoea, cramps. What is the pathogenetic mechanism that underlines these signs?A. Formation of a methemoglobinB. Formation of an oxyhemoglobinC. Formation of a carbhemoglobinD. Formation of carboxyhemoglobinE. Formation of the reduced hemoglobin

309. A 55- year-old man comes in for a check-up and complains that he is having some difficulty in «starting to urinate». Physical examination indicates that the man has a BP of 160/100 mm Hg and a slightly enlarged prostate. Which of the following medications would be useful in treating both of these conditions?A. Doxazosin B. Labetalol C. Enalapril D. PhentolamineE. Propranolol

310. Which of the following drugs is a Ca2+ blocker selective on cerebral blood vessels?A. Nifedipine B. AmlodipineC. Nimodipine D. DiltiazemE. Verapamil

Page 64: Pharmacology MCQS for 3rd yr ZSMU

311. In a patient with hypertension at regular use of an antihypertensine agent, cough has appeard. What agent can cause the given adverse effect?A. PrazosinB. Verapamil C. Clophelinum (Clonidinum) D. DichlothiasidumE. Enalapril

312. One of the following signs is not a side effect of Captopril:A. Cough B. Renal dysfunction C. Hyperkalaemia D. Hemolytic anemiaE. Fever

313. All of the following drugs produce a significant decrease in peripheral resistance except:A. Chronic administration of diuretics B. Hydralazine C.beta-blockers D. ACE inhibitorsE. Clofeline

314. Which of the folowing drugs should not be given to a pregnant, hypertensive women?A. Hydrochlorothiazide B. Propranolol C. AminophyllineD. Enalapril E. Verapamil

315. A 54-year-old man suffering from hypertension complains of severe tachycardia after administration of antihypertensive drug. Which of the following antihypertensives is most likely to cause reflex tachycardia?A. Prazosin B. VerapamileC. Hydralazine D. PropranololE. Captopril

316. A 45 year-old hypertensive male has manifested hypoglycemia after administration of antihypertensive drug. From the list of

Page 65: Pharmacology MCQS for 3rd yr ZSMU

antihypertensive drugs below select the one most likely to lower plasma sugar.A. Propranolol B. PrazosineC. Nifedipine D. CaptoprilE. Verapamil

317. A 17-y.o. man presents the symptoms of the familial hypercholesterolemia. He was prescribed an agent inhibiting enzyme 3-hydroxy-3-methylglutaryl-CoA reductase. What was the agent?A. Simvastatine B. Nicotinic acid C. Gemfibrozil D. CholestiramineE. Probucol

318. A patient 55 y.o. has suffered from the arterial hypertension for 10 years. Once in the evening he felt fatigue, acute headache, darkening in the eyes, nausea. The patient was urgently delivered to a hospital. The clinical examination revealed the increase of BP (210/110 mm Hg). The diagnosis «hypertensive crisis» was made. Administer the agents to treat hepertensive crisis.A. Anapriline and CaptoprilB. Clofeline and Furosemide C. Captopril and VerospironeD. Atenolol and VerospironeE. Anapriline and Amiloride

319. A patient suffered from the arterial hypertension for a few years. She had some accompanying diseases: chronic bronchitis with asthmatic component, chronic gastritis with increased acid-producing function. Indicate the antihypertensive drug, which is contraindicated:A. PropranololB. CaptoprilC. Reserpine D. OctadineE. All above

320. Drug of choice in hypertensive crisis of pheochromocytoma is:A. ClofelineB. CapotenC. AmlodipineD. Phentolamine

Page 66: Pharmacology MCQS for 3rd yr ZSMU

E. Drotaverine

321. A 40 y.o. patient complains of intensive heartbeats, sweating, nausea, vision impairment, arm tremor, hypertension. From his anamnesis: 2 years ago he was diagnosed with pheochromocytoma. What hypotensive agent should be administered?A. CaptoprilB. Prazosine C. DrotaverineD. NifedipineE. Platiohylline

322. A patient with II stage hypertension has been taking one of hypotensive medications. After a time AP decreased, but the patient started complaining of flaccidity, sleepiness, indifference. A bit later he felt stomach pain. He was diagnosed with ulcer. What hypotensive medication has the patient been taking?A. FurosemideB. VerapamilC. Reserpine D. DibazoleE. Captopril

323. A patient had been suffering from hypertonic disease had been treated for a long time with the drug from the group of Rauwolfia alkaloids and began to complain of heartburn, pain in the epigastrial area and bad mood. Indicate the drug which caused these complications:A. OctadinumB. Reserpinum C. ClophelinumD. PrasosineE. Enalapril

324. A patient who had been suffering from arterial hypotension had taken a hypotensive drug, but in an hour his AP increased and 2 hours after it decreased. Indicate this antihypertensive agent:A. NifedipinumB. PrazosinumC. CaptoprilD. AnaprilinumE. Octadinum

325. A patient had been suffering from hypertonic disease accompanied by chronic bronchitis with asthmatical component. Indicate the drug which is contraindicated due to it’s action on the bronchi:

Page 67: Pharmacology MCQS for 3rd yr ZSMU

A. Anaprilinum B. CaptoprilC. PrazosinumD. NifedipinE. Dichlothiazidum

326. A doctor has administered clonidine (clophelinum) to a patient to combat hypertensive crisis. What group does the named agent belong to?A. Agonists of central alfa2 adrenoceptorsB. Antagonists of central alfa2 adrenoceptorsC. Agonists of central L1 –imidazoline receptors D. Antagonists of central L1 –imidazoline receptorsE. Non-selective agonists of central alfa2-adrenoceptors

327. A doctor has administered clonidine (clophelinum) to a patient to treat hypertensive crisis. What class of antihypertensive drugs does the named agent belong to?A. Drugs affecting the renin-angoitensin systemB. Peripheral neurotropicC. DiureticsD. Central neurotropic E. Myotropic (vasotropic) hypotensive agents

328. A woman addressed to the doctor complaining of headache, fatigue, insomnia. Examination revealed high level of AP (200/100 mm Hg). A doctor administered a highly effective drug the mechanism of action of which is the inhibition of the anginomotor center activity due to excitation of a-adrenoreceptors and I-1-imidazooline receptors of NTS zone of medulla oblongata. Indicate this drug:A. PhentrolamineB. ReserpineC. HydrochlorthiazideD. Clophelinum (clonidine) E. Captopril

329. It was observed primary shorterm increase of AP in a patient after IV administration of a hypertensive agent. Indicate this preparation:A. Clophelinum (clonidine) B. ReserpineC. PrazosineD. Anaprilin (propranololo)E. Hydrochlorthiazide (dichlothiazidum)

Page 68: Pharmacology MCQS for 3rd yr ZSMU

330. Stable arterial hypertension has developed in a patient suffering from chronic glomerulonephritis. Indicate the most effective group of drugs to treat this patient:A. GanglioblockersB. Myotropic spasmoliticsC. alfa-adrenoblockersD. beta-adrenoblockersE. Angiotensine converting enzyme inhibitors

331. A patient suffering bronchial asthma had been taking tablets which caused insomnia, headache, increased BP. what medicine can cause such complication?A. Chromolin sodiumB. Ephedrine C. Euphylline D. AdrenalineE. Isadrine

332. During a surgical operation on the patient in deep general anesthesia a danger of apnea was developed. To prevent this complication 1 ml of cytitonum was introduced to the patient intravenously, but the beneficial effect wasn’t reached.Indicate the analeptic which should be used in this situation:A. CordiamineB. Coffeine C. Bemegridum D. Sulphocamphocaine E. Corasol

333. An adult patient with COPD is being managed with several drugs, one of which is inhaled Ipratropium bromide. What is the main effect that accounts for the beneficial effects of this drug?A. Blockade of endogenous bronchoconstrictor mediatorB. Enhanced release of epinephrine from the adrenal medullaC. Inhibition of cAMP breakdown via phosphodiesterase inhibitionD. Stimulation of adenylyl cyclaseE. Prevention of antigen-antibody reactions that lead to mast cell mediator release

334. A young boy with asthma receives Montelukast. What is the main mechanism by which this drug occurs its beneficial effects?A. Prevention of antigen-antibody reactions that lead to mast cell mediator releaseB. Enhanced release of epinephrine from the adrenal medullaC. Stimulation of adenylyl cyclase

Page 69: Pharmacology MCQS for 3rd yr ZSMU

D. Blockade of receptors for pro-inflammatory and bronchoconstrictor mediators E. Inhibition of cAMP breakdown via phosphodiesterase inhibition

335. The most potent bronchoconstrictor is:A. Histamine B.Bradykinin C.Leukotriens D. Acetylcholine E. Adenosine

336. A 12 y.o. boy with history of asthma complained of cough, dyspnea and wheezing. His symptoms became so severe that his parents brought him to the emergency room. Physical examination revealed diaphoresis, dyspnea, tachycardia and tachypnea. His respiratory rate was 42/min, pulse rate was 110/ min and BP was 130/70 mm Hg. Choose the most appropriate drug to reverse the bronchoconstriction rapidly:A. MethylprednisoloneB. SalmoterolC. CromolynD. IpratropiumE. Salbutamol

337. A 12 y.o. boy with history of asthma complained of cough, dyspnea and wheezing. His symptoms became so severe that his parents brought him to the emergency room. Physical examination revealed diaphoresis, dyspnea, tachycardia and tachypnea. His respiratory rate was 42/min, pulse rate was 110/ min and BP was 130/70 mm Hg. Choose the drug most likely to provide sustained resolution of the patients symptoms?A. MethylprednisoloneB. CronomolynC. SalmoterolD. IpratropiumE. Salbutamol

338. A 35-year-old woman was admitted to the surgical department with the symptoms of acute pancreatitis: vomiting,diarrhea,bradycardia, hypotension, weakness, dehydration of the organism, amylase of the blood plasma – 128U. What agent should be used first of all?A. EphedrineB. PlatyphyllineC. Contrycal (Aprotinin)D. Drotavreine (No-spa)E. Naloxone

Page 70: Pharmacology MCQS for 3rd yr ZSMU

339. Indicate the mechanism of the emetic action of apomorphine:A. Stimulation of the parasympathetic ganglionsB. Activation of chemoceptors of trigger zone of the emetic center in medulla oblongataC. Irritation of the gastric mucusD. Increase of the tonicity of smooth musclesE. Increase of secretion of hydrochloric acid

340. A doctor has administered Metoclopramide to a 37 y.o. man with the symptoms of acute pancreatitis and severe vomitus. Indicate the mechanism of action of this drug.A. Blocks M-cholinoceptorsB. Blocks dopamine D2-receptorsC Stimulates M-cholinoceptorsD. Inhibits dopamine D2-receptors

341. Hypothalamic ventromedian nuclei stroke was diagnosed in a 50 y.o. woman. Her behavior is characterized by unsatisfied hunger,weight gain. Which drug may be used to reduce her appetite?A. EtimizolB.MazindolC. PiracetamD. LipostabilE. Caffeine

342. Methotrexate (structural analogue of the folic acid, which is competitive inhibitor of the dihydrofolatreductase) is prescribed for treatment of the malignant tumor.On which level does Methotrexate hinder synthesis of the nucleic acids?A. ReplicationB. ReparationC. ProcessingD. TranscriptionE. Mononucleotide synthesis

343. A patient with the ulcer of the stomach with the increased acid-producing function was administered an agent capable to inhibit essentially 100% of gastric acid secretion in a single daily dose. What agent has been used?A. Atropine B. Ranitidine C. FamotidineD. OmeprazoleE. Platiphylline

Page 71: Pharmacology MCQS for 3rd yr ZSMU

344. A patient with a severe arthritis will be placed on long-term therapy with Indomethacin. Which of the following drugs is the most apropriate choice to administer as an add-on (adjunct) to prevent gastric ulcers caused by this NSAID?A. MisoprostolB. CelecoxibC. NimesulideD. CimetidineE. Meloxicam

345. A liquidator of a breakdown at a nuclear power plant who was irradiated complained about vomiting that occurs all of a sudden. What medication should be prescribed?A. MetoclopyramideB. ReserpineC. De-NolD. AtropineE. Aeron

346. The alternative usage of Dichlotiazide, Etacrinic acid and Lasix did not cause a marked diuretic effect in the patient with developed peripheral edema. The aldosterone level in the plasma is increased. Indicate which medicine should be prescribed:A. AmiloridB. SpironolactoneC. MannitD. ClopamidE. Urea

347. A doctor administered Allopurinol to a 37-year old man with the symptoms of gout. What pharmacological action of Allopurinol ensures therapeutical effect?A. By increasing Uric Acid excretionB. By inhibiting leukocyte migration into jointC. By inhibiting Uric Acid synthesisD. By increasing Uric Acid biotransformationE. By general anti-inflammatory effect

348. Which of the following enzymes is responsible for the ultimate formation of uric acid, which contributes to hyperuricemia and the pathophysiology of chronic and acute gout?A. 5’-LipoxygenaseB. Xanthine oxidaseC. PhospholipaseD. Cyclooxygenase-1E. Cyclooxygenase-2

Page 72: Pharmacology MCQS for 3rd yr ZSMU

349. Which of the following diuretics promote Ca2+ reabsorption?A.VerospironeB.LasixC.AmilorideD.IndapamideE.Hydrochlorothiazide

350. There is a pregnant woman in a delivery room that on term of gestation should already have labors. The uterus cervix is not opened at inspection. Indicate an agent for acceleration of uterus cervix ripening and for stimulation of parturition activity:A. ProgesteroneB. Prostin E2 (Dinoprostone) C. EstroneD. Testosterone E. Ergometrine

351. In connection with disturbances of contractility of myometrium in postpartum period, the parturient woman had uterine bleeding. What drug should administer in this situation?A. Sodium oxybutyrateB. Ergotamine C. SalbutamolD. DinoprostE. Atropine

352. Examination of a patient revealed hyperkaliemia and hyponatremia. What diuretic was administred?A. Verospirone B. FurosemideC. HydrochlorthiazideD. BumetanideE. Ethacrinic acid

353. A patient with chronic cardiac insufficiency has been treated with cardiotonic drugs and a thiazide diuretic, but in spite of it there are still edemata and risk of ascites. What medication should be prescribed to amplify diuretic effect of the applied drugs?A. AmilirideB. FurosemideC. ManitoleD. Spironolactone E. Clopamide

Page 73: Pharmacology MCQS for 3rd yr ZSMU

354. A 35 y.o. patient who often consumes alcohol was treated with diuretics. There appeared serious muscle and heart weakness, vomiting, diarrhea. AP – 100/60 mm Hg, depression. This condition is caused by intensified excretion with of: A. CalciumB. PotassiumC. ChlorineD. PhosphatesE. Sodium

355. To a pregnant woman with a history of habitual abortion, a Vitamin containing therapy was given. Select out vitamin that promotes pregnancy preservation:A. TocopherolB. Pyridoxal phosphateC. Folic acidD. RutinE. Cyanocobalamine

356. A 34 y.o. woman has arrived to the obstetrician department because of threatened (habitual) abortion. What tocolytic agent is required to administer to the patient?A. FolliculinB. Fenoterol (Partusisten) C. EthinylestradiolD. Gonadotropin menopauseE. Estradiol

357. There is an inhibited coagulation in the patient with the bile ducts obstruction, bleeding due to the low level of absorption of a vitamin. What vitamin is in deficiency?A. A B. DC. ED. KE. Carotene

358. A 2 y.o. child has got intestinal dysbacteriosis, which results in hemorrhagic syndrome. What is the most probable cause of the hemorrhage in the child?A. Vitamin K deficiencyB. PP hypovitaminosisC. Activation of tissue tromboplastinD. HypocalcaemiaE. Fibrinogen defiiciency

Page 74: Pharmacology MCQS for 3rd yr ZSMU

359. Inhibition of leukopoeisis is observed in a 43 y.o. roentgenologist. The amount of leukocytes – 3,5x109/1. Specify the agent to be used for correction of leukopoisis.A. FerroplexumB. HemostimulinumC. CyanocobalaminumD. Ascorbic acidE. Pentoxylum

360. Antidote for heparin is:A. Protamine sulfateB. EDTAC. Vicasole D. DeferoxamineE. Ascorbinic acid

361. Indicate the mechanism of the heparin action:A. Inhibition of synthesis of the clotting factors in the liverB. Inhibition of thrombinC. Inhibition of the conversion of prothrombin to thrombinD. Activation of profibrinolysinE. Causes proteolysis of plasminogen molecules to plasmin

362. Indicate the clinical use of cyanocobalamine:A. LeucopoeniaB. Malignant megaloblastic anemiaC. Acute leucosisD. GoutE. Thyrotoxicosis

363. Indicate the mechanism of antiplatelet (antithrombotic action) of aspirin:A. Inhibition of PDE that promotes the accumulation of cAMPB. Stimulation of adenilat cyclase in thrombocytesC. Increase of prostacyclin synthesisD. Block of adenosine uptakeE. Inhibition of cyclooxygenase and prevention of synthesis of thromboxane

364. Indicate the drug used locally for the arrest of hemorrhages from the small vessels.A. Calcium chlorideB. Contrical

Page 75: Pharmacology MCQS for 3rd yr ZSMU

C. AmbenD. ThrombinumE. Vikasol

365. Indicate the mechanism of the streptokinase action:A. Inhibition of synthesis of the clotting factors in the liverB. Inhibition of thrombinC. Inhibition of the conversion of prothrombin to thrombinD. Activation of profibrinolysinE. Causes proteolysis of fibrin

366. A patient 65 years, was delivered to the hospital in connection with thromboembolism of pulmonary artery, which developed after physical exertion and required intense therapy. Choose the drug of choice for this patient:A. AspirinB. DipiridamoleC. AmbciximabD. StreptokinaseE. Clopidogrel

367. The anticoagulant activity of Neodicumarin can be potentiated by all of the following drugs EXCEPT:A. Rifampicin.B AspirinC PentoxifyllineD CimetidineE Disulfiram

368. Examination of a 43 y.o. anephric patient revealed anemia symptoms. What is agent should be prescribed?A. Epoetinum (Epogen) B. Vitamin B12C. VicaslumD. Folic acidE. Fercovenum

369. A 27 y.o. woman suffers from hypochromic anemia in connection with significant blood loss after thyroidectomy. What drug should be administered to the patient? A. Cyanocobolamine (Vitamin B12)B. CoamidC. Folic acidD. PentoxilE. Ferrum-lek

Page 76: Pharmacology MCQS for 3rd yr ZSMU

370. During a competition indirect proofs of utilization of the forbidden drug erythropoietin are found in the sportsmen-skiers by results of the dope-control. Define, what is not typical for erythropoietin?A. It is preoduced by recombinant technologiesB. It decreases level of erythrocytes in bloodC. It is synthesized in kidney at conditions of hypoxiaD. It stimulates erythropoiesisE. It can be used for treatment of anemia on a background of chronic renal failure

371. To the patient suffering from stenocardia and taking isosorbide mononitrate, has been given an agent with antiaggregative activity. Indicate this agent.A. Anaprilin (Propranolol)B. Acetylsalicylic acidC. NifedipineD. NitroglycerinE. Validol

372. The patient with myocardial infarction had been receiving each 6 hours an anticoagulant IV. In a few days, he has developed bleeding gums, nasal bleeding; erythrocytes in urine have appeared. What drug was introduced to the patient?A. Aminocapronic acidB. NeodicumarinC.VicasolD. Calcium gluconateE. Heparine

373. Symptoms of internal bleeding developed at a patient of gynecologic department. What agent should be given for inhibition of fibrinolysis and arrest of hemorrhage?A. Calcium chlorideB. DicinoneC. VicasolD. ContricalE. Fibrinogen

374. A 38 y.o. woman has been on a slim diet consisted of 2 cups of coffee without sugar, 3 crackers from white bread, 2 eggs or 2 sausages a day for a month. She lost 5 kg her weight but developed headaches, often nasal bleedings, bleeding sickness of gums, shaking

Page 77: Pharmacology MCQS for 3rd yr ZSMU

of teeth, shelling of skin and shedding of hair. What vitamin drug is expedient in this case?A. Ascorbinic acidB. Cyanocobalaminum C. Folic acidD. Vicasolum (Menadione)E. Retinol acetate

375. A 32 y.o. woman visited gynecologist with the complaints of the usual abortions during the first months of pregnancy and the disorders of the menstrual cycle. Examination revealed the signs of the sexual infantility and impairment of ovulation in the ovaries. A doctor suggested the diagnosis of pituitary insufficiency. Administer the hormonal drug to treat this patient:A. CorticotrophinB. GlibenclamideC. Oxitocin

D. Gonadotropin menopausticumE. Pituitrin

376. A patient had been suffering from atopic bronchial asthma for a long time. Last time he was treated with Prednisolon because of worsening of his state. Long treatment with Prednisolon caused the development of adverse effects: edemas, increase of glucose concentration in the blood, increased excitability and sleeplessness. Indicate hormonal drug, which should be administered for prevention of this complication:A. Androcur (Cyproterone acetate)B. RetabolileC. GlibenclamideD. Tocoferol acetasE. Corticotropine

377. Determine the drug of choice for hyperprolactinemia:A. BromocryptineB. L-Dopa C. Oxitocin D. PItuitrinE. Estradiol

378. Glucocorticoids are contraindicated in all cases except:A. Organ transplantsB. Peptic ulcersC.HypertensionD. Diabetes MellitusE. Tuberculosis

Page 78: Pharmacology MCQS for 3rd yr ZSMU

379. The corticosteroid without any glucocorticoid activity is:A. Aldosterone B. Fludrocortisone C. DOCA (desoxycorticosteron acetas)D. Cortisol E. Triampicinolone

380. The following states are adverse reactions of oral contraceptives except:A. Tromboembolism B. Weight gain C.Carbohydrate intolerance D. Galactorrhea E. Hyperlipidemia

381. A woman who has been taking an oral contraceptive (estrogen + progestin) for 3 years is diagnosed epilepsy and started on Phenytoin. Which of the following states is the most probable consequence of adding Phenytoin?A. Reduced contraceptive efficacyB. Thromboembolism from the estrogen component of the contraceptiveC. Increased risk of craniofacial abnormalities in the fetusD. Aplastic anemia, requiring stopping both drugs immediately E. Seizures from increased Phenytoin clearance

382. A 32 y.o. man has complained of a headache, fatigue, thirst, pains in the spine and joints for the last 2 years. Clinically it was observed disproportional enlargement of hands, feet, nose, superciliary arches. He noted that he needed to buy 3 times bigger shoes. What is the main reason of such disproportional enlargement of different parts of the body?A. Increased sensitivity of the tissues to growth hormoneB. Increased sensitivity of the tissues to insulinC. Cartilaginous tissue proliferation under growth hormone influenceD. Joint dystrophy developmentE. Joints chronic inflammation development

383. A patient has been taking glucocorticoids for a long time. When the preparation was withdrawn he developed the symptoms of disease aggravation, decreased AP and weakness. What is the reason of this condition?A. SensibilizationB. Appearance of adrenal insufficiency C. Cumulation

Page 79: Pharmacology MCQS for 3rd yr ZSMU

D. Hyperproduction of ACTHE. Idiosyncrasy

384. A 2-year-old child experienced convulsions because of lowering Ca2+ions concentration in the blood plasma. Which treatment will be the most useful?A. Parathyroid hormoneB. InsulinC. Calcitrine D. Calcium chlorideE. Parathyroid hormone + calcium chloride

385. A 17 y.o. female suffers from tachycardia at rest, weight loss, excessive sweating, exophtalmos, and irritability, elevated Thyroxine in her serum. What treatment should be provided to the patient?A. ThyrotropinB. CalcitoninC. ReserpineD. Mercazolil (Methimazole)E. Prednisolone

386. Many therapeutic insulins are often modifications of ‘regular’ insulin. The modifications include substituting some amino acids in the protein using recombinant DNA technology, conjugating insulin with NPA (neutral protamine Hagedorn), or combining it with zinc. For all these insulins, which of the following statement is the one common result of such changes?A. Enablingadministration by either SC or IV routesB. Prevention of cellular K+ uptake C. Stimulation of pancreatic insulin synthesisD. Modification of onsets and durations of action E. Elimination of allergic responses

387. Taking oral contraceptives containing sex hormones inhibits endogenous pituitary secretion. From the list of hormones below select the one when secretion is inhibited while taking oral contraceptives containing sex hormones.A. Follicle-stimulating hormone B. Oxitocine C. PituitarineD. Growth hormoneE. Thyroid-stimulating hormone

388. At a 56 y.o. patient that complaints on thirst and frequent urination it was established diagnosis of diabetes mellitus and was

Page 80: Pharmacology MCQS for 3rd yr ZSMU

given an agent – Butamide (Tolbutamide). What is the mechanism of action of this agent?A. Promotes glucose utilization by peripheral tissuesB. Stimulates beta-cells islets of Langerhans C. Suppresses beta-cells isles of LangerhansD. Facilitate glucose transport through cellular membranesE. Inhibits glucose absorption in the intestine

389. Diagnosis of diabetes mellitus was established to a 68 y.o. patient that complains on thirst and frequent urination and was administered an agent – Metformin. What is the mechanism of action of this agent?A. Promotes glucose utilization by peripheral tissues B. Stimulates beta-cells isles of LangerhansC. Suppresses beta-cells isles of LangerhansD. Inhibits glucose transport through cellular membranesE. Inhibits glucose absorption in the intestine

390. Testosterone and its analogs increase volume of skeletal muscles that make this drug useful for the treatment of myodystrophy. Interaction with what cellular substrate determines this action?A. Membrane receptorsB. RibosomesC. Nuclear receptors D. ChromatinE. Protein-activator of transcription

391. A 35 y.o. woman that has been treated for a long time because of chronic polyarthritis is complaining on rising of AP, redistribution of fat tissues, disturbances of menstrual cycle. With what drug administrations these complain are related to?A. BeclomethasoneB. Phenylbutazone (Butadion)C. MeloxicamD. Prednisolon E. Diclofenac-natrium

392 The 32 y.o. woman was taking a contraceptive Tri-Regol for a long time. What does not belong to the adverse effects of this drug?A. ThrombosisB. PyelonephritisC. Weight gainD. Hepatotoxicity E. Osteoporosis

Page 81: Pharmacology MCQS for 3rd yr ZSMU

393. For the prevention of undesirable pregnancy the doctor–gynecologist has recommended to the 24 y.o. woman a peroral contraceptive drug, which should be applied immediately after the sexual intercourse. Specify this drug.A. RigevidonB. TrizistonC. Levonorgestrel (Postinor) D. EstradiolE. Proginova

394. A 27 y.o. woman who from time to time uses peroral contraceptives has addressed to the doctor with a question on composition of these agents. What steroid substances are included to postcoital peroral contraceptives?A. GonadotropinsB. Progestins C. AndrogensD. Anabolic steroidsE. Glucocorticoids

395. A 27 y.o. woman has arrived to the obstetrician department because of threatened (habitual) abortion. What hormonal agent is required to administer to the patient?A. Folliculin

B. Progesterone C. EthinylestradiolD. Gonadotropin menopauseE. Estradiol

396. Many kinds of pathology (inflammation, pulmonary edema, shock of different origin) are accompanied by increasing of vascular permeability. What agent from the named drugs can be used for counteraction to this at any of the named kinds of pathology?A. Dimedrolum (Diphenhydramine)B. Acid acetylsalicylicC. IndomethacinD. Prednisolone E. Beclomethasone

397. Continious taking of a drug can result in osteoporosis, erosion of stomach mucous membrane, hypokaliemia, retention of sodium and water, reduced content of corticotropin in blood. Name this drug:A. DigoxinB. HydrochlorothiazideC. ReserpineD. Indometacin

Page 82: Pharmacology MCQS for 3rd yr ZSMU

E. Prednisolone 398. A patient with infectious mononucleosis has been taking glucocorticoids for two weeks. He was brought into remission, but he fell ill with acute attack of chronic tonsillitis. What action of glucocorticoids caused this complication?A. Immunosuppressive B. Anti-inflammatoryC. AntitoxicD. AntishockE. Antiallergic

399. A 3 y.o. child has convulsions as s result of lowered concentration of Ca2+ in blood plasma. What hormone should be administered concomitantly with calcium agent?A. CalcitrinB. Parathyroid hormone (Parathormone) C. MiacalcicD. L - TyroxineE. Prednisolone

400. A 45 y.o. woman suffers from Cushing’s syndrome – steroid diabetes. Biochemical examination revealed: hyperglycemia, hypochloremia. Which of the undermentioned processes is the first to be activated? 2007A. GlycogenolesisB. GlycolysisC. Gluconeogenesis D. Glucose reabsorptionE. Glucose transport to the cell

401. Examination of a 60 y.o. patient revealed hyperglycemia and glucosuria. A doctor administered him a medication for internal use. What medication is it? A. OxytocinB. Glibenclamid C. FurosemideD. PancreatineE. Corglycon

402. A patient with collagenesis has been taking prednisolone for a long time. Hypokaliemia development caused spastic pain of skeletal muscles. What medication should be used in order to correct potassium exchange? A. NoshpaB. DithylinumC. Panangin

Page 83: Pharmacology MCQS for 3rd yr ZSMU

D. ThyrocalcitoninE. Diazepam

403. Parents of a 10 y.o. boy consulted a doctor about extension of hair-covering, growth of beard and moustache, low voice. Intensified secretion of which hormone must be assumed?A. CortisolB. OestrogenC. SomatotropinD. Testosterone E. Progesterone

404. Arachidonic acid is metabolized by two main pathways: cyclooxygenase and lipooxygenase. The latter, initially involving 5’- lipooxygenase, is responsible for the synthesis of one of the following substances. Choose the correct answer.A. LeukotriensB. Prostacyclin C. ProstaglandinsD. ThromboxanesE. Platelet-activating factor

405. A 23 y.o. man suffers from allergic seasonal coryza caused by the ambrosia blossoming. What medicine stabilizing adipose cells can be used for prevention of this disease?A. PhencarolB. DiazolineC. DimedrolD. TavegylE. Ketotifen

406. A woman with an allergic disorder experiences significant bronchoconstriction and urticaria, and histamine is a main mediator in these responses. Which of the following drugs may pose extra risk for this patient – not because it has any bronchoconstrictor effects, but because it quite effectively releases histamine from mast cells?A. AtropineB. NeostigmineC. PropranololD. PancuroniumE. d-Tubocurarine

407. A 30 y.o. woman with a 3-year history of chronic renal failure requiring dialysis consents to transplantation. A donor kidney becomes

Page 84: Pharmacology MCQS for 3rd yr ZSMU

available. She is given Cyclosporine to prevent rejection. Which of the following states is the most probable adverse effect of the drug?A. GIT ulcerationB. SeizuresC. HepatotoxicityD. NephrotoxicityE. Bone marrow depression

408. Which of the following drugs is non-sedative H1 histamine receptor antagonist?A. Diprazinum (Promethazine)B. Dimedrolum (Diphenhydramine)C. SuprastineD. LoratadineE. Tavegil (Clemastine)

409. A 22 y.o. woman has a runny nose and itching eyes from a bout with the common cold. Dimedrolum (Diphenhydramine) provides symptomatic relief. Indicate the mechanism of action that is most likely associated this the named drug.A. beta-adrenoceptors blockadeB.adrenergic stimulationC. Histamine (H1) receptor blockadeD. MAO inhibitionE. Muscarinic receptor blockade

410. An asthma patient has symptoms flare-ups during hay fever season.He visits the localsuperstore and buys an over-the-counter antihistamine/allergy remedy containing Dimedrolum (Diphenhydramine). After a few days of using it, his breathing becomes worse. You examine him and conclude that what the patient viewed as the allergy cure was actually the cause of the problems. Which of the following is the most likely mechanism by which the Dimedrolum worsened this patient condition?A. Enhancing metabolic clearance of other asthma agents (lowering their serum levels)B. Drying the airways, increasing mucus viscosityC. Releasing histamineD. Causing bronchoconstriction by releasing more Ach in the airwaysE. Blocking the endogenous bronchodilator effects of circulating epinephrine

411. A 25 y.o. man has had a kidney transplant. He receives Cyclosporine as part of the immunosupressant regimen. What is the main mechanism of Cyclosporine’s immunosupressant effects?

Page 85: Pharmacology MCQS for 3rd yr ZSMU

A. Inhibits calcinurenin and IL-2 synthesis that is necessary for B and T cell proliferation B. Inhibits antigen-activated lymphocytes, reduces responsiveness of T-lymphocytes to IL-1, reduces IL-2 production by lymphocytes and monocytesC. Directly inhibits B and T lymphocyte proliferationD. Directly destroys proliferating B and T lymphocyte proliferationE. Blocks the CD3 site on T lymphocytes, blocks all T cell functions

412. Which of the following agents is the drug of choice for treating anaphylaxis?A. EpinephrineB. NorepinephrineC. Atropine sulfateD. Dimedrolum (Diphenhydramine)E. Theophylline

413. A 21 y.o. man with asthma has what is described as ‘aspirin sensitivity’ and experiences severe bronchospasm in response to even small doses of the drug. The most likely explanation for this is that the aspirin.A. Induces hypersensitivity of H1 receptors on airway smooth musclesB. Suppresses airway inflammatory processesC. Blocks synthesis of endogenous prostaglandins that have bronchodilator activityE. Induces hypersensitivity of muscarinic receptor of airway smooth muscles

414.A doctor administered a patient with allergic dermatitis a H1_histamine blocker as a part of complex treatment. Name this medication: A. HydrocortisoneB. Cromolin sodiumC. PrednisoloneD. AdrenalineE. Loratadine

415. Dimedrolum (diphenhydramine) is administered to a patient with urticaria to reduce itching rashes on the skin. What mechanism provides its effeciency in this case?A. Competitive blockade of H1-histamine receptors B. Inhibition of synthesis of histamineC. Suppression of release of histamineD. Acceleration of histamineE. Blockade of H2- histamine receptors

Page 86: Pharmacology MCQS for 3rd yr ZSMU

416. What drug should be administered to the patient who suffers from rashes due to allergic reaction accompanied by reddening, edema, and strong itch of skin, which causes sleeplessness?A.NitrazepamumB. Dimedrolum (Diphenhydramine) C. Chlorali hydrateD. Natrii oxybutyrate (oxybate sodium)E. Phernobarbitalum

417. The patient with allergic rhinitis has taken antihistamine drug PO. In half an hour the patient felt dryness in mouth, retardation and sleepiness. Indicate this drug.A. PhernobarbitalumB. Diazolinum (Mebhydrolin)C. Dimedrolum (Diphenhydramine) D. DiazepamumE. Paracetamolum

418. A 40 y.o.parient is hospitalized with exprssed Quinke’s edema after stung of a bee. Indicate an antihistaminic drug to be introduced without sedative and hypnotic action.A. Suprastinum (Choloropyramine)B. Phencarolum (Quifenadine)C. Diazolinum (Mebhydroline)D. Tavegilum (Clemastine)E. Dimedrolum (Diphenhydramine)

419. A girl was treated with antibiotic from the group of semisynthetic Penicillins due to acute bronchitis. On the 3 rd day of treatment allergic dermatosis has developed. Indicate an antiallergic drug which should be administered t the patient.A. Suprastinum (Chloropyramine) B. LevamisoleC. Aspirin (Acetylsalicylic acid)D. CyclosporinE. Dexamethasone

420. A 21 y.o. patient has been diagnosed with allergic dermatitis. The doctor has administered to him complex therapy including the blocker of H1-histamine receptors. Indicate this drug.A. Cromolin sodium (cromoglicic acid)B. Diprazinum (promethazine)C. PrednisoloneD. AdrenalineE. Hydrocortisone

Page 87: Pharmacology MCQS for 3rd yr ZSMU

421. Indicate the group of antiallergic agents which Loratadine belongs to:A. GlucocorticoidsB. Membrane stabilizersC. Antagonists of leucotriene receptorsD. Blockers of histamine receptors E. Blockers of serotonine receptors

422. Treatment by anti-inflammatory drugs was administered to the patient with rheumatic endocarditis. After a while hyperglycemia was developed in him. What group of drugs is capable to provoke such side effect?A. Nonsteroid anti-inflammatory agentsB. d- PenicillamineC. MethotrexatD. SulfasalazineE. Glucocorticoids

423. The immunodepressive effect of Prednisolone is cause by:A. Inhibition of protein synthesis due to activation of gluconeogenesis B. Inhibition of collagen synthesisC. Activation of synthesis of inhibitors pf proteasesD. Inhibition of synthesis of mucopolysaccharidesE. Diminution of activity of plasmin

424. A 35 y.o. patient, suffering from bronchial asthma, is hospitalized in a state of anaphylactic shock. What drug is necessary to introduce first of all as first aid?A.Dimedrolum (diphenhydramine)B. Adrenaline C. Chromoglicic-acidD. SalbutamolE. Ephedrine

425. Anaphylactic shock has developed at the patient after intracutaneous test on sensitivity to Pencillin. The doctor had administered a drug which eliminated bronchospasm and arterial hypotension. Indicate this drug:A. Mesatonum (Phenylephine)B. NoradrenalineC. Adrenaline D. Atropine

Page 88: Pharmacology MCQS for 3rd yr ZSMU

E. Salbutamol

426. Many kinds of pathological states (inflammation, pulmonary edema, shock of different origin) are accompanied by violation of permeability of vessels. Which of the listed below drugs can be used for elimination of the reaction at any of the termed states?A. Aspirin (acetylsacylic-acid)B. IndomethacinumC. Dimedrolum (diphenhydramine)D. Prednisolone E. Beclomethasone

427. Indicate the drug which is the most expedient to use topically to treat allergic dermatitis?A.Sea buckthorn oil (Oleum Hippophae)B. Solution of furacilinum (nitrofuran)C. Liniment of methylsalicylateD. Ointrent of inchyolum (ichthammol)E. Hydrocortisone ointment

428. The doctor has administered a glucocorticoid Synaflanum (fluocinoline) in the form of ointment to the patient with dermatitis. What pharmacolodical effect is not characteristic for this drug?A.Magnification of hyperemia B. Diminution of itchC. Diminution of edemaD. Deceleration of wound repairE. Inhibition of skin immune response

429. Fluocinolone acetonide is administered to the patient, suffering from allergic cheilitis for smearing of lipline and mucous of the lips. Indicate the group which this drug belongs to?A. Antragonists of histamine H1 receptorsB. Antragonists of histamine H2 receptors C. Glucocorticoids D.Membrane stabilizersE. Antragonists of luekotriene receptors

430. A doctor administered Cromolyn sodium (cromoglycate sodium) to the patient suffering from bronchial asthma to prevent attacks. Indicate the principle of action of this drug:A. Decrease of concentrations of immunoglobulinsB. Binding of free histamineC. Stabilization of membranes of mast cells D. Inactivation of histamine

Page 89: Pharmacology MCQS for 3rd yr ZSMU

E. Blockade of histamine receptors

431. A 45 y.o. woman suffers from allergic seasonal coryza caused by Ambrosia blossoming. What drug from the group of stabilizers of mast cells can be used for prevention of the disease?A. Ketotifen B. PhencarolC. TavegilD. DimedrolE. Diazoline

432. Ketotifenum is administered to a patient with bronchial asthma. Indicate mechanism of action of this antiallergic drug:A. Blocks H2-histamine receptorsB. Blocks H1-and H2-histamine receptorsC. Inhibits synthesis of histamineD. Blocks H1- histamine receptors E. Activates enzymes which cleave histamine

433. What agent from the group of adrenomimetics is the most effective for local use in allergic rhinitis?A. Mesatonum (Phenylphine)B. Naphthyzinum (Naphazoline)C. FenoterolD. SalbutamolE. Ephedrine

434. A 23 y.o. patient took Levomycetinum (chloramphenicole) for a long time without doctor permission. The patient examination revealed leucopenia. What drug should be administered for stimulation of leucopoiesis?A. Pentoxylum B. MethotrexateC. MercaptopurineD. CyancobalamineE. Prednisoline

435. The patient with chronic infectious disease requires treatment with non specific immunostimulant agent. Indicate this drug.A. MercaptopurineB. Methyluracil C. AzathioprineD. ActinomycinE. Cyclophosphane

Page 90: Pharmacology MCQS for 3rd yr ZSMU

436. A 48 y.o. woman suffering from exacerbation of chronic pnenumonia requires treatment with immunostimulant agent. Indicate this drug.A. SulfocamphocainumB. Thymalinum C. Biseptol (co-trimoxazole)D. Dimedrolum (diphenhydramine)E. Levamisole

437. Indexes of immune response are worsened in a patient during chemotherapy of malignant tumour. What drug should be administred in this state?A. Thymalinum B. CyancobalamineC. Iron preparationsD. PrednisoloneE. Acetylsalicylic acid

438. A drug from the group of immunostimulant, which is an analogue of natural biogenic substrate and almost does not exert side effects is administerered to a 4 y.o. child. Indicate this drug.A. ThymalinumB. ProdigiosanumC. Interferon D. PyrogenalumE. Levamisole

439. It is necessary to administer an antihistamine agent to a woman suffering from seasonal vasomotor rhinitis, who works as a dispetcher on the railway. Indicate the drug to treat the patient:A. Cromolyn sodium (cromoglycate sodium)B. Diprazinum (promethazine)C. PrednisoloneD. TelfastE. Hydrocortisone

440. A patient suffering from chronic generalized parodontitis requires the treatment with immunostimulant agent, which also possesses antihelmintic activity. Indicate this drug.A. Pyrantel pamoateB. Seeds of pumpkin C. CycloferonD. CyclosporineE. Levamoisole

Page 91: Pharmacology MCQS for 3rd yr ZSMU

441. Which vitamin or nutrient, also an ingredient in some prescription medications for severe, refractory acne vulgaris, is “highly” teratogenic and should not be administered pregnant women?A. Vitamin AB. Vitamin B6C. Vitamin CD. Vitamin EE. Vitamin K

442. Fat-soluble vitamins, compared with their water-soluble forms, generally have a greater potential toxicity. They are:A. Involved in more essential metabolic pathwaysB. Involved in less essential metabolic pathwaysC. Avidly stored by the body D. Capable of dissolving membrane phospholipidsE. Administered in larger doses

443. The patient should be told to avoid taking supplemental Vitamin B6 (pyridoxine) if he is being treated with one of these drugs:A. Digoxin B. Haloperidol C. Niacin D. Levodopa (Carbidopa) E. Phenytoin

444. You have a patient who has been consuming extraordinarily large amounts of alcohol for several years. He goes into acute withdrawal and manifests nystagmus and bizarre ocular movements and confusion (Wernicke’s encephalopathy). Although this patient’s alcohol consumption pattern has been accompanied by poor nutrient intake overall, you specifically manage the encephalopathy by administering one of these drugs:A. Vitamin AB. Folic acidC. Vitamin B12 (Cyanocobalamine)D. Vitamin E (alfa-tocopherol)E. Vitamin B1 (Thiamine)

445. Indicate the main feature of Trisaminum (Trometamol) that Sodium bicarbonicum is unable:A. It is available for taking orallyB. It is available for IV infusionC. It penetrates into the cell, binding to hydrogen ions outside and inside the cell

Page 92: Pharmacology MCQS for 3rd yr ZSMU

D. It does not penetrate into the cell that is why it eliminates extracellular acidosis onlyE. It increases BP

446. You are doing summer volunteer work at health clinic in a very poor region of the world. A 19-y.o. woman is diagnosed Vitamin D-resistant rickets. Apart from a high-dose of Vitamin D and oral phosphate, an additional therapeutic approach might be used. What agent should you choose?A. EstrogenB. HydrochlorothiazideC. Folic acidD. CalcitriolE. Vitamin B12

447. What drug from the list below is used to inhibit pancreas function?A. UrokinaseB. PancreatinC. AlteplaseD. Contrycal (Gordox)E. Lydazum

448. The vitamin agent, deficiency of which can cause sterility in experimental animals, and de applied to complex treatment of female infertility. Specify this vitamin.A.Vitamin AB. Vitamin KC.vitamin B1D. Vitamin DE. Vitamin E

449. A 45 y.o. patient was admitted to the hematologic department with acute anemia: RBCs-1,5 x 10.12/L, Hb _80g%, colour index 1,3. Hyperchromic anemia was diagnosed.Which drug should be administered for treatment of this disease?A. HemostimulineB. FerroplexumC. Cyanocobalamine D. Ferrum-lek E. Folics acid

450. Which of the acids below decreases permeability of connective tissue structures, possesses antioxidant activity due to ability to be transformed from the oxidized from into reduced and on the contrary?A. Aspirin (acetylsalicylic acid)

Page 93: Pharmacology MCQS for 3rd yr ZSMU

B. Hydrochloric acidC. Mefanamic acidD. Ascorbic acid E. Aminocaproic acid

451. For synthesis of the basic substances of connective tissue (mucopolysaccharide and collagen) an essential agent is:A. Nicotinic acidB. Ascorbic acid C. Folic acid D. Salicylic acidE. Acetylsalicylic acid

452. Radiation therapy is performed to the patient. What vitamin drug with antioxidant properties is necessary to administer to increase stability of tissues in this case?A. Thiamine chlorideB. Vitamin B6C. Ascorutinum D. CyanocobalamineE. Folic acid

453. A dermatologist diagnosed red flat herpes in a 28 y.o. patient and administered him under the locus of lesion 30 injections of a vitamin drug, which participates in oxidative phoshorylation. Specify this drug:A. Nicotinic acid B. Thiamine bromideC. Ascorbic acidD. Vitamin B6E. Ergocalciferol

454. The patient who was treated by a vitaminic drug for prophylaxis of vasospasms of the brain has developed complaints of the unpleasant sensations related to taking of this medicine: reddening of the face and the upper half of a trunk, giddiness, sense of flush of blood to a head. For what drug the specified side effects are characteristics?A. Tocofecol acetateB. NicotinamidumC. Thiamine bromideD. Nicotinic acid E. Calcium pangamate

Page 94: Pharmacology MCQS for 3rd yr ZSMU

455. Diarrhea, dementia and dermatitis are observed at the patient. What vitaminic drug should be included into a complex treatment? A. ThiaminB. CyanobalamineC. Nicotinamide (vitamin PP) D. Calcium pantothenateE. Cyanocobalamine

456. 55 y.o. woman suffers from hemaralopia (disturbance of vision in darkness). What vitamin drug should be recommended her first of all?A.RiboflavinB. Tocoferol acetateC. PyridoxineD. Ascorbic acidE.Retinol acetate

457.Which vitamin promotes growth and development of epithelial cells, including epidermal ones?A. RetinolB.ErgocalciferolC. Ascorbic acidD. Nicotinic acidE. Lipoid acid

458. 37 years old patient suffers from hyperkeratosis, disturbance of vision in darkness, frequent infectious deseases. What vitaminic drug should be administered for treatment?A. PyridoxinB. Retinol acetate C. RiboflavinD. ErdocalciferolE. Tocoferol acetate

459. The patient had been taking vitamin D for a long time for treatment of rickets. The phenomena of intoxication have developed: depression of appetite, nausea, headache, fatigue, disorders of sleep, increase of body temperature, changes in urine – hyaline cylinders, protein, leucocytes. What vitaminic drug should be administered to weaken arisen symptoms?A. Vikasolum (Menadione) B. Vitamin B12C. Vitamin A (Retinol acetate) D. Vitamin PPE. Riboflavin

Page 95: Pharmacology MCQS for 3rd yr ZSMU

460. Parasthesia, xeroderma and sticking out fontanel are observed at a 6 months child under the treatment by a vitaminic drug. Specify this drug:A. Retinol acetateB. PyridoxineC. RiboflavinD. Ergocalciferol E. Tocoferol acetate

461. What of the listed below vitaminic drugs possesses expressed radioprotective property?A. Ergocalciferol B. Thiamine chlorideC. RiboflavinD. Tocoferol acetate E. Folic acid

462. What enzymatic drug is used with the purpose of reduction of density and rising of permeability of connective tissue structures?A. AmylaseB. LipaseC. CocarboxylaseD. CholinesteraseE. Lidase

463. According to clinical indications a patient was administered pyridoxal phosphate. What processes is this medication intended to correct?A. Dissemination of purine nucleotideB. Synthesis of purine and pyrimidine basesC. Transamination and decarboxylation of aminoacids D. Protein synthesisE. Oxidative decarboxylation of kenotic acids

464. Examination of a man who hadn’t been consuming fats but had been getting enough carbohydrates dermatitis, poor wound healing, vision impairment. What is probable cause of metabolic disorder?A. Lack of oleic acidB. Low caloric value of dietC. Lack of palmitic acidD. Lack of vitamins PP, HE. Lack of linoleic acid, vitamins A, D, E, K

Page 96: Pharmacology MCQS for 3rd yr ZSMU

465. Removal of gall bladder of a patient has disturbed processes of Ca2+ absorption through the intestinal wall. What vitamin will stimulate this process?A. PPB. B12C. CD. KE. D3

466. The patient with the diagnosis of focal tuberculosis of the upper lobe of a right lung receives isoniazid in complex therapy. After a while the patient started to complain on muscle weakness, desensitization of the skin, disturbance of vision and coordination of movement. What vitamin agent should be used for removing of these phenomena? A. Vitamin CB. Vitamin B6 C. Vitamin ED. Vitamin DE. Vitamin B12

467. During hypertensive crisis magnesium sulfate has been introduced to the patient and after that AP fell down. Which drug introduction will remove this side effect of magnesium sulfate?A. Sodium bromideB. Potassium bromideC. Trilon BD. Ferrum-LekE. Calcium chloride

468. A patient with the abscess of the cut wound applied to the traumatologist. The wound was washed with 3% Hydrogen Peroxide to be cleaned from the pus. Foam was not observed. What caused inefficiency of the drug?A. Pus in the woundB. Shallow woundC. Inherited insufficiency of catalaseD. Low concentration of H2O2E. Inherited insufficiency erythrocyte’s phosphatdehydrogenase

469. A 23 y.o. patient addressed to an ophthalmologist with complaints of eye discomfort, discharge of purulent exudate, disorders of vision. Specify the antiseptic to rinse the eyes:A. Lugol’s solutionB. Potassium permanganate

Page 97: Pharmacology MCQS for 3rd yr ZSMU

C. Ammonium solutionD. Silver nitrateE. Chlorhexidinum

470. For the preparation of a patient’s burn skin surface a certain medication was used. Its antiseptic action is provided by free oxygen that segregates in presence of organic substances. Choose the right answer:A. FuracilinB. Boric acidC. ChlorhexidineD. Potassium permanganate E. Sodium bicarbonate

471. A 57 y.o. patient with varicose dilation of veins develops the trophic ulcer of the leg. The bacteriological examination of the ulcer discharge has revealed Staphylococcus infection. Determine the antiseptic in the form of ointment from the group of detergents for local treatment of the ulcer:A. AethoniumB. Brilliant greenC. FuracillinumD. Potassium permanganateE. Ethacridinum lactas

472. All antiseptics possess all following properties except:A. Selective antimicrobic actionB. Versatile antimicrobic actionC. Are not introduced parenterallyD. Highly toxic for humanE. Bactericidal action

473. 70% solution of ethyl alcohol is used for surgeon’s hands cleaning before operation. Explain the mechanism of action of the drug:A. Protein dehydration of microbes protoplasmB. Interaction with hydroxilic groups of microbes’ enzymesC. Interaction with aminogroups of protoplasm proteins of microbesD. Blockade of sulfhydryl groups of enzymesE. Oxidation of organic components of microbe’s protoplasm

474. Which antiseptic is used for surgical tools cleaning?A. FuracillinumB. 70% solution of ethyl alcoholC. 95% solution of ethyl alcohol

Page 98: Pharmacology MCQS for 3rd yr ZSMU

D. EthoniumE. Resorcinum

475. A doctor used 5% spirituous solution of Iodine for operation field cleaning. Indicate its mechanism of action:A. Inhibition of dehydrogenaseB. Interaction with amino groups of microbes proteins that promotes to their denaturationC. Dehydration of protoplasm’s proteinsD. Binding to enzymes’ sulfhydric groupsE. Formation of albuminates

476. Choose the drug for a 6 month old child with mycoplasma infection:A. TetracyclineB. StreptomycinC. Clotrimazole D. ErythromycinE. Penicillin

477. Which antibiotic is not used in myasthenia gravis? A. TetracyclineB. GentamycineC. ClarithromycinD. CeftriaxonE. Azithromycin

478. A 20 y.o.patient was hospitalized to the surgical department because of infection caused by blue pus bacillus (Pseudomonas aeruginosa), which is sensitive to penicillin antibiotics.Indicate which of the given penicillins has a marked activity on the Pseudomonas aeruginosa?A. BenzylpenicillinB. MethicillinC. PhenoxymethylpenicillinD. Carbenicillin disodiumE. Oxacillin

479. The patient with pneumonia was treated with antibiotics for a long period. After the treatment the patient complains of frequent and watery stools, abdominal pain. What is the reason of intestinal disorder?A. Hereditary enzyme defectB. Bacteria toxins influenceC. Intestinal dysbacteriosis development

Page 99: Pharmacology MCQS for 3rd yr ZSMU

D. Antibiotics toxic influence on the GITE. Allergic reaction

480. Purulent endometritis developed in a woman after delivery. Treating with antibiotic inhibitors of murein synthesis was ineffective. Wide spectrum bactericidal antibiotic was administered to her.In 6 hours t0 rapidly increased up to 400C with shiver. Muscle pains have appeared. BP dropped down to 70/40 mm Hg. Oliguria has developed. What is the reason for the development of this condition?A. Toxic effect of preparationB. Endotoxic shockC. Anaphylactic shockD. BacteremiaE. Internal bleeding.

481. Choose the drug of choice for cholera prophylaxis:A. ChloramphenicolB. Procaine Penicilline C. Doxycycline D. ErythromycinE. All above

482. All of the following clinical indication may require a combination of antibiotics (rather than a single agent) except:A. Treatment of mixed infectionsB. Treatment of gonorrheaC. Treatment of tuberculosisD. Treatment of meningitisE. Treatment of bacterial endocarditis

483. A 50-year old woman who had underwent chemotherapy for lymphoma a week ago was brought to the emergency room. She had a fever of 40.3oC and was confused. Respiration was rapid and BP was 75/40. She was neutropenic. Gram’s strains of the urine and sputum are negative. Which of the following actions is the most beneficial to the patient?A Send a clinical sample to the laboratory for identification and then administer an appropriate antibiotic. B. Administer a combination such as Clindamycin and an aminoglycosideC. Administer a broad-spectrum antibiotic like TetracyclineD. Administer ClindamycinE. Administer Tobramycinum

Page 100: Pharmacology MCQS for 3rd yr ZSMU

484. A 60-year-old alcoholic male with poor dental hygiene is to have his remaining teeth extracted for subsequent dentures. He has mitral valve stenosis with mild cardiac insufficiency and is being treated with captopril, digoxin and furosemide. The dentist decides that his medical history warrants prophylactic antibiotic therapy prior to the procedure and prescribes:A. Vancomycin B. AmoxicillinC. TetracyclineD. Co-trimoxazoleE. Imipenem

485. A 35-year-old man has an infection with Legionella. Assuming no contraindications, which of the following drugs should you choose?A. PenicillinB. GentamycinC. ErythromycinD. ChloramphenicolE. Tetracycline

486. Which of the following drugs is the most effective agent in the treatment of Rickettsia,Mycoplasma,and Chlamydia infections?A. PenicillinB. GentamycinC. ErythromycinD. ChloramphenicolE. Tetracycline

487. Streptomycin and other aminoglycosides inhibit bacterial protein synthesis by binding with one of the following substances.A. 30S ribosomal particlesB. DNAC. mRNAD. Peptidoglycan units in the cell wallE. RNA polymerase

488. A child who previously was healthy develops bacterial meningitis. Assuming no specific contraindications, which of the following drugs will you prescribe?A. ErythromycinB. TetracyclineC. DoxicyclineD. CeftriaxoneE. Procaine penicilline

Page 101: Pharmacology MCQS for 3rd yr ZSMU

489. A patient being treated for springtime allergies with Loratidine develops an upper respiratory problem. He receives an antibiotic and develops a cardiac arrhythmia. What kind of antibiotic do you think it was?A. CephaclorB. OxacillinC. Cephalotin D. ErythromycinE. Amoxicillin

490. Which of the following drugs is both penicillinase-resistant and effective by oral administration?A. MethicillinB. CarbenicillinC. CeftriaxoneD. Amoxicillin plus clavulanic acidE. Procaine penicillin

491. The penicillin that is effective against Proteus and Pseudomonas aeruginosa is:A. AmpicillinB. AmoxiclavC. AmoxycillinD. CarbenicillinE. Oxacilline

492. Chronic ulceration would most likely occur after administration of:A. CarbenicillinB. Clindamycin C. Choramphenicol D. GentamycinE. Doxycyclinum

493. Most serious adverse effect of penicillins is:A. Skin rashes B. Jarish hexheimer reactionC. Anaphylaxis D. ConvulsionsE. Urticaria

494. The diagnosis of sepsis has been made to a patient. It was decided to use a drug from the group fluoroquinolones. Determine this drug.

Page 102: Pharmacology MCQS for 3rd yr ZSMU

A. ErythromycinB. CiprofloxacinC. GentamycinD. TetracyclinesE. Amikacin

495. To treat an exacerbation of chronic pyelonephritis the doctor has prescribed a drug from the group fluoroquinolones. What is the mechanism of antimicrobial action of fluoroquinolones?A. Inhibition of DNA-gyraseB. Inhibition of synthesis of peptidoglycan C. Inhibition of protein synthesis by binding to 30S ribosomesD. Inhibition of protein synthesis by binding to 50S ribosomesE. Increasing of bacterial wall permeability

496. A patient suffers from severe postoperative pseudomonades infection. Which antibiotic should be administered in this case?

A.Amikacin sulfate B.Benzylpenicillin C.ErythromycinD.CephazolinE.Doxycyclin

497. An antibiotic with ability to penetrate to bone tissues and cumulate there was administered to 25 y.o. patient with osteomyelitis. After 3 weeks of using it the patient felt much better. Indicate the drug:A. Chloramphenicol (Levomycetin)B. OxacillinC. AmpicillinD. PenicillinE. Lincomycin

498. Determine the drug that able to penetrate to bone tissue and bone marrow to treat bone infections:A. GentamicinB. Neomycin

C. ClindamycinD. OleandomycinE. Erythromycin

499. Staphilococcus aureus resistent to methicillin and other beta-lactam antibiotics has revealed in bacteriological blood test of a 27 y.o. woman with puerperal sepsis. Which drug should be administered in this case?

Page 103: Pharmacology MCQS for 3rd yr ZSMU

A. VancomycinB. TetracyclinC. CefazolineD. GentamycinE. Amphotericin

500. A patient with diminished hearing has severe bacillary infection. Which pharmacological group is contraindicated to the patient?A. TetracyclinesB. MacrolidesC. TetracyclinesD. AminoglycosidesE. Fluoroquinolones

501. Infectious agent determined by lab tests is known to be sensitive to third generation cephalosporins. Choose the drug for treatment:A. CefazolinB. CephalothinC. CephalexineD. CefoperazoneE. Ceftriaxone

502. A patient on antimicrobial therapy develops the following signs and symptoms that ultimately are found to be drug-induced:A cough, dyspnea, and pulmonary infiltrates; neutropenia and bleeding tendencies.Which of the following can most likely cause this patients symptoms?A. AmoxicillinB.CiprofloxacinC. AzithromycinD.Furadoninum(Nitrofurantoin)E. Isoniazid

503. Which of the following drugs is primarily used in amebiasis and leishmaniasis as well as anaerobic bacterial infections?A.Co-Trimoxazole (Biseptol)B. CiprofloxacinC. AzithromycinD. MetronidazoleE. Carbenicillin

504. A patient requires an antibiotic that is the most effective against P.aeruginosa. Which of the following drugs is the quinolone of choice?A. Ofloxacin

Page 104: Pharmacology MCQS for 3rd yr ZSMU

B. CiprofloxacinC. LomefloxacinD. NorfloxacinE. Enoxacin

505. Which property or mechanism of action is shared by the penicillins, the cephalosporins, and Amphotericin B?A. Act through various mechanisms on cell walls or membranes of susceptible organismsB. Contraindicated in immunocompromised patientsC. Interact with many drugs by inducing their hepatic metabolismD. Interact with many drugs by inhibiting their hepatic metabolismE. Nephrotoxicity precludes use in patients with impaired renal function

506. A 37 y.o. woman complains of itching in the vulval area. Hanging-drop examination of the urine reveals trichomonads. Which of the following drugs is preferred for the treatment of trichomoniasis?A. AmoxicillinB. CiprofloxacinC. AzithromycinD. Furadoninum (Nitrofurantoin)E. Metronidazole

507. The drug of choice for chloromphenicol resistant typhoid fever is:A. CefaclorB. CefotaximeC. Ciprofloxacin D. CefazolineE. Ampicilline

508. A patient has been diagnosed gonorrhea after bacterioscopy of the smear from urethra. Taking into account that agents of choice for the treatment of gonorrhea are fluoroquinolones the patient is necessary to administer:A. Erythromycin B. CiprofloxacinC. Furasolidone D. ClaritromicinE. Rifampicin

Page 105: Pharmacology MCQS for 3rd yr ZSMU

509. A patient with pneumonia has intolerance of antibiotics. Which combined sulfonamide preparation should be administered for the treatment of this patient?A. SulfadimidineB. SulfadiazineC. Co-trimoxazoleD. SulfasalasineE. Sulfamethoxazole

510. Maximum endocrinal side (gynecomastia, impotence) effects are seen with:A. Ketoconazole B. FluconazoleC. Myconazole D. NystatinE. Ampicilline

511. Primary mechanism of action of Zidovudine is:A. Proliferation of B and T cellsB. Inhibition of reverse transcriptaseC. Activation of macrophagesD. Stimulation of lymphocytesE. Blockade of leukotrien receptors

512. A 35-year-old man under the treatment for pulmonary tuberculosis has acute-onset of right big toe pain, swelling, and low-grade fever. The gouty arthritis was diagnosed and high serum uric acid level was found. Which of the following antituberculosis drugs is known for causing high uric acid level?A. RifampicinB. Aminosalicylic acidC. ThiacetazoneD. PirazinamideE. Cycloserine

513. A patient with tuberculosis is being treated with Isoniazid. He develops paresthesias, muscle pain, and unsteadiness. Which vitamin needs to be given in supplemental doses in order to reverse these symptoms – or used from the outset to prevent them in high-risk patients?A. Vitamin AB. Vitamin CC. Vitamin KD. Vitamin B1 (thiamine)E. Vitamin B6 (pyridoxine)

Page 106: Pharmacology MCQS for 3rd yr ZSMU

514. A 59 y.o. man is diagnosed tuberculosis. Before prescribing a drug regimen, you take a careful medication history because one of the drugs commonly used to treat tuberculosis induces microsomal cytochrome P-450 enzymes in the liver. Which drug is it?A. RifampinB. Vitamin B6 (pyridoxine)C. IsoniazidD. PirazinamideE. Ethambutol

515. A patient with active tuberculosis is being treated with Isoniazid and Ethambutol as part of the overall regimen. Which of the following statements is the main reason for including the Ethambutol?A. To facilitate entry of Isoniazid into the mycobacteriaB. To facilitate penetration of the blood-brain barrierC. To slow renal excretion of IsoniazidD. To retard the development organism resistanceE. To retard absorption after IM injection

516. As a part of a multidrug attack on a patient’s infection with Mycobacterium tuberculosis, a physician plans to use an aminoglycoside antibiotic. Which of the following drugs is the most active against the tubercle bacillus and seems to be associated with the fewest problems with resistance or typical aminoglycoside-induced adverse effects?A.Amikacin B. KanamycinC. NeomycinD. StreptomycinE. Tobramycin

517. Amantadine (Midantanum), used prophylactically against influenza A2, is thought to act by:A. Preventing production of viral capsid proteinB. Preventing uncoating of viral DNAC. Preventing virion releaseD. Preventing penetration of the virus into the host cellE. Causing lysis of infected host cell by release of intracellular lysosomal enzymes

518. A 30 y.o. man is HIV-positive with a cluster-of-differentiation-4 (CD4) count of 200/mm3. Within 2 months he develops a peripheral white blood cell count of 1000/mm3 and a hemoglobin of 9.0 mg/dL. Which of the following drugs most likely caused the hematological abnormalities?A. Rimantadine

Page 107: Pharmacology MCQS for 3rd yr ZSMU

B. MidantanumC. AcyclovirD. FoscarnetE. Zidovudine

519. A patient with AIDS is treated with a combination of agents, one of which is Zidovudine. This drug exerts its main effects by inhibiting:A. Viral proteasesB. Viral particle assemblyC. RNA synthesisD. Nonnucleoside reverse transcriptaseE. Nucleoside reverse transcriptase

520. Which antituberculous agent acts as the competitive antagonist of para-aminobenzoic acid?A. IsoniasidB. EthambutolC. Sodium Paraaminosalicilas (PAS)D. RifampicinE. Kanamycin

521. Hyperuricemia is due to inhibition of uric acid secretion in kidney: gout can occur as a result of adverse effect of:EthosuximideCycloserinePyrazinamideRifampicinAmpicillin

522. A patient was diagnosed with active focal pulmonary tuberculosis. What drug should be prescribed in the first place?A. Isoniazid B. CyclocerineC. EthionamideD. EthoxideE. Sulfalen

523. Pyrantel pamoate is effective in both:A. Amoebiasis and trichuriasisB. Taenia solium and ascariasisC. Amoebiasis and strongyloides D. Enterobius and ascariasis E. Strongyloides and enterobius

Page 108: Pharmacology MCQS for 3rd yr ZSMU

524. Chloroquine in malaria acts on:A. Erythrocytic cycleB. Exoerythrocytic cycleC. Liver D. SchizontsE. All above

525. Flushing occurs after alcohol ingestion in patients taking:A. Chingamine B. PenicillinC. TetracyclineD. ChloramphenicolE. Metronidazole

526. Which of the following antimalarial drugs is relatively safe in pregnancy:A. PrimaquineB. Co-trimoxazole C. ChloridineD. ChingamineE. Mefloquine

527. Megaloblastic anaemia is a consequence of all drugs except:A. TrimethoprimB. MethotrexateC. Amoxycillin D. PyrimethamineE. Co-Trimoxasole

528. Intraluminal amoebicide of choice isA. Metronidazole B. Diloxanide furoate C. Chloroquine D. TetracyclineE. Penicillin

529. To a patient with ascariasis it was given a drug that has also action on immune system and is used as immunomodulator. What is the name of this drug?A. PyrantelB. Piperazine adipinateC. Naphthammone (Bephenium)D. LevamisoleE. Phenasalum (Niclosamide)

Page 109: Pharmacology MCQS for 3rd yr ZSMU

530. A 58-year-old woman has just returned from a trip to Southeast Asia. Over the past 24 hours she developed shaking, chills, and to of 40.5oC. A blood smear reveals Plasmodium vivax. Which of the following agents should be used to eradicate the extraerythrocytic phase of the organism?A. ChloroquineB. PrimaquineC. PyrimethamineD. TetracyclineE. Quinacrine (Acrichinum)

531. A young boy presents the infestation with Taenia saginata (tapeworm). Which of the following drugs is the most appropriate drug to administer?A. MebendazoleB. Niclosamide (Phenasalum)C. ChloroquineD. TetracyclineE. Penicillin

532. In a patient it was revealed mixed helmintic invasion: intestinal cestodes and liver trematodes. What agent from anthelmintic drugs should be presvribe?A. AlbendazoleB. PraziquantelC. LevamisoleD.Pyrantel E. Piperazine

533. Mother addressed to the pediatrician with the child who complainted of strong itches in the region around the anus, pain intensified at night. After investigation of feaces enterobiasis has been diagnosed. Indicate the drug, which should be administered.A Piperazine adipinate B. TrichlorophenumC. PhenasalumD. Ditrazinum E. Aminoacrichinum

534. Indicate the antimalarial agent, which is active against paraerythrocytic forms of Plasmodium.A. AmodiachinumB. ChingaminumC. Galochinum

Page 110: Pharmacology MCQS for 3rd yr ZSMU

D. Hydroxychlorochinum E. Primachinum

535. For malaria prevention before journey abroad a doctor has got an agent with histoschizontocidic action. What drug did the doctor take?A. PrimaquineB. QuinineC. DoxycyclinD. Biseptol (Co-trimaxazole) E. Chloridine (Pyrimethamine)

536. Specify the drug, which is used in amebiasis of any localization of pathological process.A Metronidazolum (Trichopolum) B. ChingaminumC. Emetinum D. Chiniophonum E. Tetracyclinum

537. A woman addressed to a gynecologist in relation with large discharge from the vagina with unpleasant smell. After laboratory examination the diagnosis of trichomoniasis in this situation? A SulfadimezinumB. Metronidazolum (Trichopolum) C. ChingaminumD. Chloridinum E. Monomycinum

538. A patient visited a physician with complaints of bowel dysfunction. After laboratory examination the diagnosis of lambliasis was made. Specify the drug that should be used:A TetracyclinumB. TrichomonacidC. Metronidazolum (Trichopolum) D. Monomycinum E. Chingaminum

539. During summer vacations a student from tropical country developed tertian malaria. After recovery he turned back to Ukraine for study extension. In January an exacerbation was developed. It is known from past history of disease that drug acting on paraerythrocytic forms of plasmodium malariae was prescribed. Indicate the drug: A ChingaminumB. HalochinC. Hydroxycholoquine

Page 111: Pharmacology MCQS for 3rd yr ZSMU

D. Amodiaquine E. Primaquine

540. Indicate the alkaloid, which is used for treatment of malaria, and also possesses other pharmacological activities: decreases excitabity of the myocardium, stimulates rhythmical contractions of the uterus, side effects are noise and ringing in the ears, decrease of hearing:A Chininin hydrocholoridum B. PrimachineC. ChloridinumD. ChingaminumE. Metronidazolum

541. Indicate the drug that exerts paralyzing action on nematodes, increases tonicity and contractions of the smooth muscles of the intestine, so it is used without a purgative agent. It has low toxicity and is used mainly for ascariasis and entrobiasis:A. Pyrantale pamoate B. MebendazoleC. PrimachineD. Metronidazolum E. Praziquantel

542. A patient consulted a doctor about bowel disfunction. The doctor established symptoms of duodenitis and enteritis. Laboratory examination helped to make the following diagnosis: lambliosis. What medication should be administered? A. MonomycinB. Metronidazole C. ChingaminD. TetracyclineE. Erythromycin

543. For prevention remote relapses of 4-days malaria a 42 y.o. patient was given primaquine. On the 3rd day of the treatment with therapeutic doses of the drug patient experienced abdominal pain, cardiac pain, dyspepsia, generalized cyanosis. What is the reason of these adverse effects of the drug? A Cumulation of the therapeutic agentB. Slowing down of drug’s eacretion with urineC. Decreasing of activity of liver microsomal enzymesD. Genetic deficiency of Glucose-6-phosphate dehydrogenase E. Potentiation of the drug’s action by other therapeutic agent

Page 112: Pharmacology MCQS for 3rd yr ZSMU

544. Methotrexate (structural analogue of the folic acid which is competitive inhibitor of the dihydrofolatreductase) is prescribed for the treatment of the malignant tumor. On which level does Methotrexate hinder synthesis of the nucleic acids?A. ReplicationB. ReparationC. ProcessingD. TranscriptionE. Mononucleotide synthesis

545. Which phase of the cell cycle is resistant to the most chemotherapeutic agents, i.e. those that are classified as phase-specific?A. Go B. G1C. G2 D. ME. S

546. A cancer man receives prophylactic Allopurinol before a course of chemotherapy. Which of the following statements is the main purpose of doing this?A. Facilitate host cell detoxification of the chemotherapeutic drug, thereby reducing host cell toxicitiesB. Reduce the risk of hyperuricemia and its main consequences (renal damage, gout) that can occur with a massive cell kill C. Inhibit the potential for DNA repair, that otherwise might lead to chemotherapy failureD. Potentiate the action of a nitrosourea to bind to purine moieties in DNA strandsE. Prevent myelosupression and related blood dyscrasias

547. Which of the following statements is the main mechanism by which the Vincristine exerts its main effects?A. Alkylating DNA, causing cross-links between parallel DNA strandsB. Blocking microtubular assembly and mitosis during M-phase C. Inhibiting topoisomerase, preventing repair of DNA strand breaksD. Intercalating in DNA strands, thereby preventing DNA replication by mRNAE. Stabilizing assembled microtubular arrays, thereby preventing mitosis

Page 113: Pharmacology MCQS for 3rd yr ZSMU

548. Which of the following is the main mechanism by which the Cyclophosphamide exerts its cell killing?A. Alkylating DNA, causing cross-links between parallel DNA strands B. Blocking microtubular assembly and mitosis during M-phaseC. Inhibiting topoisomerase, preventing repair of DNA strand breaksD. Intercalating in DNA strands, thereby preventing DNA replication by mRNAE. Stabilizing assembled microtubular arrays, thereby preventing mitosis

549. Which of the following conditions is the most likely adverse response to occur as a result of the Vincristine action?A. Nephrotoxicity, renal dysfunction or failureB. Peripheral sensory and motor neuropathy C. Pulmonary damageD. AgranulocytosisE. Rhabdomyolysis

550. A patient with advanced Hodgkin’s disease is placed on combination therapy with Vincristine, Embichinum (Chlormethine), Procarbazine, and Prednisone (the so-called MOPP regimen). Which of the following procedures plays the main role of the Prednisone effects in this therapeutic plan?A. Preventing opportunistic infectionsB. Exerting direct cytotoxic actions, independent of the other drugsC. Counteracting fluid overload from chemotherapy-induced renal dysfunctionD. Counteracting hyperglycemia caused by the other agentsE. Suppressing emesis and vomiting

551. A 25-year-old-woman with choriocarcinoma is treated with very high doses of Methotrexate. You anticipate significant host cell toxicity in response to the high Methotrexate dose, and so immediately after giving the anticancer drug you administer one of the following drugs:A. Vitamin KB. Vitamin B1C. Vitamin B6D. Vitamin B12E. Folic acid

552. While reviewing charts in a general medicine clinic you see that a woman, 27 y.o. and with no history of cancer at all, is also taking Methotrexate. The drug is most likely given to manage one of the following conditions. What is it?

Page 114: Pharmacology MCQS for 3rd yr ZSMU

A. HyperthyroidismB. Asthma or emphysemaC. Clinical goutD. Myasthenia gravisE. Rheumatoid arthritis or psoriasis

553. Allopurinol should be avoided, or reduced doses of the chemotherapeutic agent, if the anticancer drug is one of the following:A. DoxorubicinB. CysplatinC. MercaptopurineD. CyclophosphamideE. Vincristine

554. A patient with Wilm’s tumor is receiving a chemotherapeutic agent that is described as acting by intercalating into DNA strands, and that is efficacious regardless of what stage of the cell cycle the tumor cell are in. Which of the following agents best fits this description?A. Dactinomycin (Actinomycin) B. Cytarabin (Cytosine arabiniside)C. MercaptopurineD. CyclophosphamideE. Vincristine

555. A cancer patient develops severe, irreversible cardiomyopathy because the maximum dose of an anticancer drug was exceeded. Which of the following drugs is most likely responsible for this patient’s symptoms?A. Doxorubicin B. CysplatinC. MercaptopurineD. CyclophosphamideE. Vincristine

556. A 45 y.o. man complains of progressive difficulty starting his stream urinating, and having to get up at least once night to urinate. Rectal examination reveals an enlarged, smooth-surfaced prostate. Prostatic serum antigen (PSA) titers are elevated.Urine flow increases,and prostate size decreases,in response to Cyproterone acetate_(Androcur) treatment. This drugs main mechanism of action involves one of the following processes.What is it?A.5±-reductase \inhibitionb.±1-adrenergic receptor blockadeC. Lowering serum Testosterone levels

Page 115: Pharmacology MCQS for 3rd yr ZSMU

D. Testosterone synthesis inhibitionE. Competition with dihydrotestosterone for intracellular androgen receptor and inhibition its binding

557. The drug used to treat Methotrexate toxicity is:A. Folic acid B. Folinic acidC. RiboflavinD. CyanocobalaminE. Vicasol

558. The following drugs are alkylating agents except:A. CyclophosphamideB. Methotrexate C. ChlorambucilD. SarcolysinumE. Myelosanum

559. The antitumoral agent from the group of antimetabolites (antagonist of the folic acid) was administered to the patient with acute leucosis. Indicate this drug.A. Methotrexatum B. FluorouracilC. MyelosanumD. MercaptopurineE.Hexestrol

560. Determine the antitumoral drug from the group of antimetabolites which is used for treatment of leucosis in children and cancer in adults:A Methotrexate B. SarcolysinumC. ColchamineD. Rubomycin E.Predrnisolone

561. A patient takes an acute, massive overdose of aspirin that, without proper intervention, will be fatal. Which of the following conditions would you expect in the advanced (late) stages of aspirin (salicylate poisoning)?A. Metabolic alkalosisB. Respiratory acidosisC. Respiratory alkalosis, then - metabolic acidosis

Page 116: Pharmacology MCQS for 3rd yr ZSMU

D. HypothermiaE. Ventilatory stimulation

562. In addition to providing symptomatic, supportive care, which of the following drugs would be a helpful adjunct to manage severe Paracetamol (Acetaminophen) poisoning?A. NaloxonB. Acetylscystein C. DiazepamD. Sodium bicarbonateE. Unitiolum

563. Which of the following conditions is the primary cause of death from massive Acetaminophen overdoses?A. Acute nephropathyB. Status epilepticusC. Status asthmaticusD. A-V conduction disturbancesE. Liver failure

564. A patient has taken a potentially lethal dose of Acetaminophen (Paracetamol). The current preferred antidotal therapy involves administration of drug that:A. Inhibits synthesis of superoxide anion radical and hydrogen peroxideB. Is rich in sulfhydryl (-SH) groups C. Alkalinizes the urine to facilitate Acetaminophen excretionD. Inhibits hepatic oxidative metabolism to inhibit formation of Acetaminophen’s toxic metabolitesE. Causes metabolic acidosis to combat the toxic metabolite’s metabolic alkalosis

565. A patient who receives a rapid IV injection of a drug develops hypocalcemic tetany. Which of the following drugs is the most likely cause it?A. Edetate Sodium (Trilon B) B. UnitiolumC. PenicillamineD. DeferoxamineE. Acetylscysteine